Емкость конденсатора это: Электрическая емкость. Конденсаторы | LAMPA

Содержание

формула для расчета электрической емкости

Конденсатор – радиоэлектронный прибор, способный накапливать и отдавать заряд. Как правило, на его корпусе дается информация о его емкости, но иногда требуется самому рассчитать этот номинал. Конденсаторами могут выступать и проводники, они также обладают определенной емкостью. Для расчета существует несколько формул емкости конденсатора, их и рассмотрим.

В чем измеряется емкость конденсатора

Что такое заряд еще проходят в школе, когда эбонитовую палочку натирают о шерстяную ткань и подносят к маленьким кусочкам бумаги. Под действием электромагнитных сил бумага прилипает к палочке. Подобный заряд накапливается в конденсаторе. Но для начала познакомимся с самим конденсатором.

Простейшим конденсатором являются две металлические пластины, разделенные диэлектриком. От качества диэлектрика зависит, как долго энергия заряженного конденсатора может сохраняться. На этих пластинах, они еще называются обкладками, накапливается разноименный заряд. Как это происходит?

Электрический заряд, а в случае с металлами это электроны, способен перемещаться под действием электродвижущей силы (э. д. с.). Подключая металлические пластинки к источнику тока, мы получаем замкнутую цепь, но разделенную диэлектриком. Электростатическое поле проходит этот диэлектрик, замыкая цепь, а электроны, дойдя до препятствия, останавливаются и скапливаются.

Получается, на одной обкладке наблюдается избыток электронов, и эта пластина имеет отрицательный знак, а на другой пластине электронов недостает настолько же, знак на этой обкладке, конечно же, будет положительным.

Вот теперь нужна для определения емкости конденсатора формула, определяющая, какой заряд способен разместится на конкретном конденсаторе.

В качестве единицы измерения в международной системе (СИ) емкость определяется в Фарадах.

Много это или мало — емкость в 1Ф? Чтобы конденсатор обладал емкостью в 1Ф, он должен содержать в себе заряд в 1К (кулон) и при этом напряжение между обкладками должно равняться 1 вольту.

Интересно. Что такое заряд в 1 кулон? Если два предмета, каждый из которых имеет заряд в один кулон разместить в вакууме на расстоянии один метр, то сила притяжения между ними будет равна силе притяжения землей тела массой в один миллион тонн.

Как и любая буквальная емкость один и тот же конденсатор может вмещать разное количество заряда.

Рассмотрим пример.

  • В трехлитровую банку входит три литра воздуха. Его хватит для дыхания, допустим, на 3 минуты. Но если воздух закачать под каким-то давлением, то емкость так и останется три литра, однако дышать можно будет дольше. Так устроен акваланг для ныряльщиков. Получается, количество воздуха в банке зависит от давления, которое в ней создается. Точно так же есть некая зависимость между различными силами, влияющими на емкость.

Формула емкости плоского конденсатора

Прежде чем узнать,

по какой формуле вычисляется емкость плоского конденсатора, рассмотрим формулу для одиночного проводника. Она имеет вид:

  • где Q – заряд,
  • φ – потенциал.

Как видно емкость конденсатора, формула которого здесь приведена, будет тем больше, чем больший заряд способен накапливаться на нем при незначительном потенциале. Чтобы легче это было понять, рассмотрим получившие широкое распространение плоские конденсаторы разных размеров.

Для получения качественного конденсатора важны любые мелочи:

  1. ровная поверхность каждой обкладки;
  2. обе пластинки по всей площади должны располагаться на одинаковом расстоянии;
  3. размеры обкладок должны быть строго идентичными;
  4. от качества диэлектрика, расположенного между пластинками, будет зависеть ток утечки;
  5. емкость напрямую зависит от расстояния между обкладками, чем оно меньше, тем больше емкость.

Теперь обратимся к

плоскому конденсатору. Формула определения емкости конденсатора несколько отличается от приведенной выше:

  • где S – площадь одной обкладки,
  • ε— диэлектрическая проницаемость диэлектрика,
  • ε0 — электрическая постоянная,
  • d – расстояние между обкладками.

Электрическая постоянная выражается числом 8,854187817×10-12.

Внимание! Эта формула справедлива только тогда, когда расстояние между пластинами намного меньше их площади.

Попробуем разобраться с каждой переменной подробнее. Площадь измеряется в м2, точнее, приводится к этой величине. А вот проницаемость диэлектрика может обозначаться по-разному.

В России это ε(также означает относительная проницаемость), в англоязычной литературе встречается ε(также означает абсолютная проницаемость), а то может и вовсе использоваться без индекса, просто ε. О том, что здесь используется диэлектрическая проницаемость диэлектрика можно понять из контекста.

Дальше идет ε0. Это уже вычисленное значение, измеряемое в Ф/м. Последняя переменная – d. Измеренное расстояние также приводится к метру. Емкость конденсатора, формула которого сейчас рассматривается, показывает сильную зависимость от расстояния обкладок. Поэтому стараются это расстояние по возможности сокращать. Почему этот показатель так важен?

Идеальными условиями для получения наибольшей емкости – это отсутствие промежутка между обкладками, чего, конечно, добиться невозможно. Чем ближе находятся разноименные заряды, тем сильнее сила притяжения, но здесь возникает компромисс.

При уменьшении толщины диэлектрика, а именно он разделяет разноименные заряды, возникает вероятность его пробоя из-за разности потенциалов на обкладках. С другой стороны, как уже говорилось, при увеличении напряжения увеличивается количество зарядов. Вот и приходится выбирать между емкостью и рабочим напряжением конденсатора.

Есть другая формула для плоского переменного конденсатора:

Здесь диэлектрическая проницаемость обозначена буквой ε, π = 22/7 ≈ 3,142857142857143, d – толщина диэлектрика. Формула предназначена для конденсатора, состоящего из нескольких пластин.

Допустимая толщина диэлектрика d также зависит от εr, чем выше коэффициент, тем тоньше можно использовать диэлектрик, тем большую емкость будет иметь конденсатор. Это был самый сложный материал, дальше будет легче.

Формула емкости цилиндрического конденсатора

Теперь поговорим о том, как найти емкость конденсатора цилиндрической формы. К ним относятся конденсаторы, состоящие из двух металлических цилиндров, вставленных один в другой. Для разделения между ними расположен диэлектрик. Формула емкости конденсатора выглядит следующим образом:

Здесь видим несколько новых переменных:

  • l – высота цилиндра;
  • R1 и R2 – радиус первого и второго (внешнего) цилиндров;
  • ln – это не переменная, а математический символ натурального логарифма. На некоторых калькуляторах он имеется.

Всегда нужно помнить, что все величины должны приводиться к единой системе, в приведенной ниже таблице указаны международные системы единиц (СИ).

Из нее видно, что все расстояния нужно приводить к метру.

Еще стоит обращать внимание на качество диэлектрика. Если толщина диэлектрика влияет только на емкость конденсатора, то его качество затрагивает сохранность энергии. Другими словами, конденсатор с качественным диэлектриком будет иметь меньший саморазряд.

Определить качество можно по числу, стоящему возле вещества, чем оно больше, тем лучше качество. Сравнение производится по вакууму, значение которого равно единице.

Формула емкости сферического конденсатора

Последнее что осталось разобрать – формулу определения емкости конденсатора,

состоящего из двух сфер. Причем одна сфера находится внутри другой. Формула имеет следующий вид:

Из приведенных переменных здесь все знакомо. Стоит обратить внимание лишь на сам конденсатор.

Кроме своей необычной формы у него есть свои особенности: внутри малой сферы никакого заряда нет, он образуется на внешней части малой сферы и внутренней части большого шара. Также заряд отсутствует и на внешней стороне внешней сферы.

Так же как и все другие конденсаторы, сферы разделены диэлектриком. Толщина и качество диэлектрика оказывают такое же влияние на емкость, как в случае с другими конденсаторами.

После того как были рассмотрены формулы, стоит испробовать их на практике. Рассмотрим, как найти емкость конденсатора каждого вида.

Примеры решения задач

Начнем с плоского конденсатора. Формула для этого вида:

Допустим, у нас есть следующие значения:

  • в качестве диэлектрика возьмем слюду толщиной 0,02 мм, ε = 6;
  • конденсатор квадратный со сторонами в 7 мм.

Определяем площадь пластин: 7×7 = 49 мм2.

Приводим к единой системе: 4,9×10-5 = 0,000049 м2. Толщина диэлектрика 0,02×10-5 = 0,00002 м. Электрическая постоянная 8,854187817×10-12.

Подставляем в формулу и высчитываем числитель: 6×8,854187817×10-12 ×4,9×10-5, сокращаем и решаем 6×49×8,854187817×10-17 = 2,603131218198×10-14.

Делим на толщину диэлектрика: 2,603131218198×10 / 2×10 = 1301,565609099×10 = 1,301565609099×10. Шесть нулей – это тысячи или приставка «микро», получается округлено 1,3 мкФ.

Возможно, при вычислении была допущена ошибка, но это не экзамен по математике. Важно понять сам метод вычисления.

Формула для цилиндрического конденсатора:

Выбираем значения:

  • l = 1 см;
  • R1 = 0,25 мм;
  • R2 = 0,26 мм;
  • ε = 2.

Подгоняем под единую систему: l — 1 см = 1×10-2 = 0,01 м; R1 – 0,25 мм = 0,0025 м; R2 – 0,26 мм = 0,0026 м.

Подставляем значения в числитель: 2×3,142857142857143×8,854187817×10-12×2×0,01 1,11×10-12. Находим знаменатель: 0,26:0,25 = 1,04.

Находим натуральный логарифм, он равен примерно 0,39. Числитель делим на знаменатель: 1,11×10-12/0,39 = 2,85×10-12.

Число с 12 нулями это приставка «пико», получаем 2,85 пФ.

Формула для сферического конденсатора:

Выбираем значения:

  • ε= 4;
  • r1= 5 см;
  • r2= 5,01 см.

Снова все подгоняем: 5 см = 0,05 м; 5,01 см = 0,0501 м. Заполняем числитель. 4×3,142857142857143×4×8,854187817×10-12×0,05×0,0501 1,11×10-12 Вычисляем знаменатель: 0,0501 – 0,05 = 0,01. Производим деление: 1,11×10-12×0,01 = 1,11×10-10

. Снова получили пикофарады, а именно 1,11 пФ.

Похожие материалы на сайте:

Понравилась статья — поделись с друзьями!

 

Емкость конденсатора — Справочник химика 21

    I. Приготовить несколько разбавленных растворов полярного вещестьа в неполярном растворителе. 2. Измерить емкость конденсатора, заполненного растворителем и каждым из приготовленных растворов. 3. Рассчитать диэлектрическую проницаемость каждого из растворов, используя табличное значение диэлектрической проницаемости растворителя, взятое из справочника при той же температуре, при которой производились измерения емкости. 4. Измерить плотности растворов всех концентраций при той же температуре, при которой были измерены емкости. 5. Рассчитать по уравнению (И,22) поляризацию растворенного веш,ества. 6. Построить график зависимости поляризации растворенного вещества от концентрации раствора и экстраполировать завпсимость до предельного разбавления. 7. Определить показатель преломления растворенного вещества и вычис лить молярную рефракцию. 8. Рассчитать по уравнению (И, 17) ди польный момент растворенного вещества. [c.99]
    Один из методов установления электрического момента диполя молекул основан на измерении диэлектрической проницаемости веществ при разных температурах. Для этого вещество в виде газа или разбавленного раствора в неполярном растворителе помещают между обкладками конденсатора. При этом емкость конденсатора увеличивается в е раз (е—диэлектрическая проницаемость). Если емкость конденсатора в вакууме обозначить С , а емкость с веществом С, то [c.156]

    Возрастание емкости конденсатора (за счет уменьшения силы электрического поля в е раз) обусловливается тем, что энергия тратится на деформационную поляризацию молекул и их ориентацию в поле. [c.156]

    Здесь Л/ —мольная масса р —плотность е —диэлектрическая постоянная (равна отношению емкости конденсатора С, между пластинами которого находится данное вещество, к емкости Со того же конденсатора, когда между его пластинами — глубокий вакуум). [c.68]

    Емкость конденсатора связывает его заряд с разностью потенциалов между обкладками. Мы рассматриваем двойной электрический слой на поверхности электрода как конденсатор и относим емкость двойного слоя и его заряд к 1 поверхности электрода Тогда связь между С — удельной емкостью двойного слоя, в — плотностью его заряда и скачком потенциала получает такой вид  [c.538]

    Генератор А р к у с — обеспечивает широкий интервал электронно-управляемых режимов дуги переменного и униполярного тока с частотой следования разрядов 20 33,3 и 100 с (для дуги переменного тока) и 10 12,5 16,6 25 и 50 С (для униполярной дуги). Фаза поджига имеет три фиксированные значения — 60, 90 и 120°. Сила тока дуги может изменяться от 1 до 16 А при наличии внутреннего реостата и до 30 А с внешним реостатом. Емкость конденсатора в режиме низковольтной искры равна 40 мкф. [c.63]

    Тела, которые совсем не проводят электрического тока (воздух, стекло, смола, сера, резина, эбонит и т. д.) или проводят его слабо, называются непроводниками электричества, или диэлектрическими. Опыты показывают, что при употреблении какого-либо твердого или, жидкого диэлектрика в качестве изолирующего вещества емкость конденсатора при прочих равных условиях больше, нежели при изоляторе — воздухе. [c.56]

    Диэлектрическая проницаемость характеризует электромагнитные свойства среды. В численном выражении величина диэлектрической проницаемости определяется отношением емкостей конденсатора в диэлектрике с и вакууме Со [c.75]

    Величина С выражает емкость конденсатора, который при разности потенциалов между обкладками Va несет заряд, равный заряду двойного слоя. [c.167]

    Жидкость II газ после продувки технологической аппаратуры и все другие сбросы газа и жидкости при работе и остановке оборудования должны собираться в продувочную емкость-конденсатор, соединенную с факелом для сжигания газа. При наличии в аппаратах и трубопроводах газового конденсата проводить продувку непосредственно в атмосферу не допускается. [c.100]

    Принцип действия датчика давления следующий. Давление на выходе исследуемого образца фиксируется уровнем жидкости в пьезометрической трубке. С изменением давления меняется уровень жидкости, что приводит к изменению емкости цилиндрического конденсатора, образованного электродами 3 -а 6. Изменение емкости конденсатора вызывает расстройку анодного контура частотного преобразователя 2, на выходе которого изменяется сигнал постоянного тока, поступающего на выход самопишущего прибора 1. В качестве самопишущего прибора использован самопишущий миллиамперметр типа Н37 с классом точности 0,5. [c.133]

    Со — емкость конденсатора в вакууме. [c.530]

    Измерение диэлектрической проницаемости растворов сводится к определению емкости конденсатора, заполненного воздухом или другим веществом с известной диэлектрической проницаемостью и исследуемым веществом. Как известно из курса физики, емкость плоского или цилиндрического конденсатора при площади обкладок 5 и расстоянии между ними с1 выражается соотношением [c.332]

    Iq — зарядный ток, обусловленный емкостью конденсатора, чисто реактивный ток  [c.532]

    Возможно и другое определение понятия диэлектрическая проницаемость , вполне равноценное приведенному определению. Именно диэлектрическую проницаемость вещества можно определить как число, выражающее отношение величины емкости конденсатора, заполненного данным веществом, [c.398]

    Определение диэлектрической проницаемости проводили в растворах криоскопического бензола. Непосредственно измеряемой величиной была емкость конденсатора, наполненного криоско-пическим бензолом — Со и наполненного раствором асфальтенов в криоскопическом бензоле — С. Диэлектрическую проницаемость рассчитывали по формуле [c.34]

    Емкость конденсатора, заполненного жидкостью, измеряли автоматическим мостом переменного тока Р-589. Во всех случаях наблюдали следующее распределение диэлектрических проницаемостей (по убыванию) раствор металлопорфириновых комплексов, извлеченных из асфальтенов раствор асфальтенов раствор асфальтенов, лишенных порфиринов. Сравнение величины Ае/С асфальтенов (рис. 14) с этой же величиной самых полярных фракций смол, изученных автором [169], показывает, что полярность асфальтенов гораздо выше. Этим можно объяснить высокую меж-фазную активность асфальтенов. [c.34]


    Диэлектрическую проницаемость рассчитывают, исходя из электростатической емкости конденсатора, а не по величине индуцированного заряда Р. [c.316]

    Кроме величин ей или момента е /4л = е8 двойной слой можно также характеризовать его электрической емкостью С, которая эквивалентна емкости конденсатора плош адью 1 см и определяется соотношением [c.151]

    Эту величину можно сравнительно легко и точно измерить методом, подобным тому, который используют при измерении емкости конденсаторов. Теория Гуи—Чепмена позволяет рассчитать С путем дифференцирования поверхностного заряда по потенциалу диффузного слоя, исходя, например, из (5.31). Для оценки С можно воспользоваться более простым выражением (5.32), справедливым для малых Фо- Это дает [c.151]

    Таким образом, емкость конденсатора пропорциональна диэлектрической проницаемости [c.332]

    Затем опускают конденсатор в исследуемую жидкость, обеспечивая достаточную глубину погружения, и фиксируют бюкс с жидкостью с помощью поворотного столика и крепежного винта. Измеряют емкость конденсатора. [c.333]

    Второй эффект, обусловливающий возрастание емкости конденсатора, проявляется для полярных молекул, т. е. молекул, обладающих постоянным дипольным моментом [г. Электрическое поле стремится ориентировать молекулы соответствующими концами диполя в направлении положительной и отрицательной обкладок конденсатора. Этот эффект называют ориентационной поляризацией Р . Она тем значительнее, чем больше р,. Ориентационная поляризация зависит от температуры, так как нагревание, усиливая тепловое движение молекул, препятствует их ориентации. [c.189]

    Возрастание емкости конденсатора определяется полярностью и поляризуемостью молекул, т. е. зависит от постоянного ц и наведенного р, д дипольных моментов молекул. Изменение Р с температурой обусловлено изменением Р , т. е. в конечном счете величиной х. [c.189]

    Оно выражает линейную зависимость поляризации вещества от абсолютной температуры. Если измерить емкость конденсатора с изучаемым веществом (а следовательно, и е) при разных температурах п построить график зависимости Р от 1/Т, то получится прямая. По наклону прямой (В) можно найти дипольный момент (в О) из выражения [c.190]

    Для определения диэлектрической проницаемости среды можно воспользоваться измерением емкости конденсатора, которая ирямо пропорциональна диэлектрической проницаемости. Емкость конденсатора с плоскопараллельными электродами определяется по уравнению [c.95]

    Соотношение емкостей конденсатора, заполненного разными диэлектриками, равно соотношению их диэлектрических проницаемостсп [c.95]

    Добиться максимального отклонения стрелки вращением лимба отсчет по стрелочному прибору. 6. Увеличить чувствительность прибора поворотом рукоятки Чувствительность по часовой стрелке. При этом показание миллиамперметра должно уменьшаться, а чувствительность возрастать. 7. Вновь настроить рукояткой отсчег Сд иа максимальное иоказание миллиамперметра. 8. Увеличить по возможности отклонение стрелки прибора и опять добиться максимального отклонения стрелки прибора рукояткой компенсация потерь . 9. Произвести отсчет по барабану и лимбу. Измеряемая емкость равна сумме показаний на лимбе и на барабане. Полученную величину умножить на показание переключателя множитель . 10. Измерить емкость конденсатора (в пикофарадах), заполненного эталонной жидкостью с известным значением диэлектрической проницаемости и исследуемой. И. Измерить емкость конденсатора с эталонной и с исследуемой жидкостью нри четырех-няти температурах. 12. Вычислить дипольный момент по уравнениям (И,15) и (11,16). [c.96]

    К рассматриваемому классу относятся также зонды, состоящие из двух небольших металлических пластин, образующих конденсатор и соединенных с наружным регистрирующим устройством Так как твердые частицы и ожижающий агент имеют различные диэлектрические свойства, то емкость конденсатора зависит от концентрации твердых частиц между пластинами. В отличие от описанного выше светового датчика, являющегося, по существу, двухпоаициопным устройством, мощность сигнала от емкостного датчика связана (хотя и несколько неопределенно) с массой твердых частиц между пластинами, что позволяет непрерывно измерять их концентрацию. [c.124]

    Дипольные моменты обычно определяют экспериментально, измеряя диэлектрическую проницаемость е веществ при различных температурах. Если вещество поместить в электрическое поле, создаваемое конденсатором, то емкость последнего возрастает в г раз, т. е. г = с1со (где Со и с — емкость конденсатора в вакууме и в среде вещества). [c.71]

    На пилотной установке непрерывного действия колонного типа (рис. 97) можно получать дорожные, строительные, кровельные и специальные битумы разных марок, изучать влияние природы сырья и параметров режима окисления на свойства битумов. Ее основные аппараты резервуары для сырья емкостью 2 л (диаметр 210 мм, высота 260 мм) трубчатый подогреватель из стальных труб длиной 1500 мм, внутренним диаметром 6 мм с электрообогревом окислительная колонна диаметром 80 мм, высотой 1000 мм с тремя боковыми отводами для отбора проб битума, ])асположепными па выоте 300, 600 и 900 мм от днища колонны напорная емкость конденсатор-холодильник для конденсации и охлаждения паров и газообразных продуктов окисления приемник для конденсата (отдува) приемник для битума (на схеме пе показан). [c.277]

    Во время проведения эксперимента контролируется изменение толщины пленки нефти на поверхности металлической сферы, опущенной ниже границы раздела нефть — вода. Исследуемые жидкости помещены в металлический стакан. Измерение емкости конденсатора, одной обкладкой которого служит поверхность сферы, а другой — граница раздела нефть — вода, производится емкостным мостом. Течение пленки возникает под действием архимедовых сил и отражается в изменении средней толщины пленки при выдавливании ее в основной объем нефти. При малом зазоре между обкладками конденсатора связь измеряемой величины емкости со средней толщиной пленки выражается следующей формулой [10]  [c.52]

    Величину Сд можно найти с помощью уравнения (ХХУП1. 16) путем измерения емкости конденсатора с воздухом (Свозд) и со стандартной жидкостью, точное значение диэлектрической проницаемости которой известно. [c.332]

    Таким образом, определение диэлектрической проницаемости сводится к измерению емкости конденсатора с воздухом (Свозд), со стандартной жидкостью (С]) (например, бензолом, диэлектрическая проницаемость которого составляет 2,27 при 25 °С) и, наконец, с исследуемой жидкостью (раствором) — С. [c.332]

    После настройки контуров присоединяют параллельно анодному контуру конденсатор, емкость которого необходимо измерить (Сх). Условие резонанса будет нарушено, и миллиамперметр покажет отсутствие переменного тока. Вращая ручку настройки переменного конденсатора, вновь восстанавливают состояние резонанса. Тогда, зная емкость переменного конденсатора до и после присоединения С , по разности находят емкость исследуемого конденсатора. Обычно для этой цели специально градуируют переменный конденсатор и строят градуировочную кривую зависимости емкости конденсатора от угла поворота ручки настройки. По этому графику находят измеряемую емкость. В качестве индикатора резонанса в таком приборе может быть использован оптический индикатор настройки, (например лампа 6Е5С). Наименьшая ширина теневого сектора на светящемся экране индикатора отвечает максимальному току в колебательном контуре (резонанс). [c.334]


3.1.9 Конденсатор. Электроёмкость конденсатора. Электроёмкость плоского конденсатора

Видеоурок 1: Емкость конденсатора — Физика в опытах и экспериментах


Видеоурок 2: Емкость плоского конденсатора. Классификация конденсаторов

Лекция: Конденсатор. Электроёмкость конденсатора. Электроёмкость плоского конденсатора

Для определения понятия конденсатора, необходимо воспользоваться всеми знаниями по поводу проводников и диэлектриков, поскольку и те, и другие одновременно используются для изготовления конденсаторов.


Ёмкость конденсатора

Во время изучения понятия конденсаторов будем использовать термин «уединенный конденсатор». Он значит, что данное проводящее тело находится вне поля действия других тел. Рассмотрим проводник, у которого имеется какой-то определенный потенциал «фи». Данный потенциал является пропорциональным к величине заряда проводника. Данный коэффициент принято обозначать 1/С. В данном случае величина, что находится в знаменателе, является электрической ёмкостью имеющегося проводника.


Если рассматривать некоторый объемный шар в виде проводника, то его ёмкость можно определить по следующей формуле:


Эпсилон нулевое — это электрическая постоянная.

Судя из данной формулы, можно сделать вывод, что емкость сферического проводника зависит исключительно от внешнего диэлектрика, а также от радиуса сферы, то есть её размера. То есть, чем больше сфера, тем больше её емкость. Иными словами можно сказать, что ёмкость — это некая тара, которая вмещает в себя величину заряда.

Итак, 


Ёмкость — это некая ФВ, позволяющая определить величину заряда, необходимую для изменения потенциала проводника на 1 В.

Ёмкость измеряется в фарадах (Ф).

Самым главным для нас сферическим проводником является Земля. Посчитать её емкость достаточно просто. Если принять радиус за 6400 км, а окружающую диэлектрическую среду мы знаем — это воздух, то получим:


Плоский конденсатор

Самым простым примером для исследования конденсаторов являются плоские конденсаторы. Структура плоского конденсатора достаточно проста. Он состоит из двух металлических плоскостей (обкладок), которые параллельно расположены друг к другу, и располагаются на некотором расстоянии. Между данными пластинами имеется диэлектрик.

Самым простым примером плоского конденсатора является тот, у которого в виде диэлектрика воздух, то есть Ɛ = 1.

Обе обкладки имеют противоположный заряд -q, +q. Схематически конденсатор изображают следующим образом:


Следует отметить, что поле в конденсаторе показывается линиями, выходящими из положительно заряженной пластины, и входящими в отрицательно заряженную пластину.

Для определения его ёмкости следует воспользоваться следующей формулой:


Как уже говорилось раньше, ёмкость зависит исключительно от геометрических размеров конденсатора, а также от диэлектрика между пластинами.

Лабораторная работа №6 определение электрической емкости конденсатора контрольные вопросы

Лабораторная работа №6

Определение электрической емкости конденсатора.

Контрольные вопросы.

1. Конденсатор в переводе — сгуститель. По какой причине прибору дано такое странное название?

Ответы: 1. Т.к. он предназначен для накопления и удержания магнитной энергии.

2. Т.к. он предназначен для накопления и удержания на своих обкладках равные по значению и разные по знаку электрические заряды + Q и -Q.

2. В чем сущность указанного метода определения емкости кон­денсатора?

Ответы: 1. Заряд, накопленный на конденсаторе, прямо пропорционален силе тока в замкнутой цепи, куда включен конденсатор, а число делений на измерительном приборе зависит от емкости конденсатора.

2. Магнитная энергия, накопленная на конденсаторе, пропорциональна силе тока в цепи, куда включен конденсатор известной емкости. По числу делений на измерительном приборе в пропорции определяют емкость неизвестного конденсатора.

3. Объяснить, можно ли соотношение С =прочесть так: емкость конденсатора прямо про­порциональна его заряду и об­ратно пропорциональна напряжению между его обклад­ками?

Ответы: 1. Да. Потому что емкость конденсатора – это коэффициент пропорциональности между зарядом и напряжением между обкладками конденсатора.

2. Нет. Потому что емкость конденсатора прямо пропорциональна напряжению на его обкладках и обратно пропорциональна его заряду.

3. Нет. Потому что ёмкость С = не зависит от напряжения, а заряд при Uonst зависит от С.

4. Почему емкость конден­сатора постоянна?

Ответы: 1. Т.к. это физическая константа.

2. Т.к. эта величина определяет заряд, который нужно сообщить одной его пластине, чтобы вызвать повышение напряжения между пластинами на 1В.

5. От чего зависит емкость простейшего конденсатора? Запишите формулу этой емкости.

Ответы: 1. С = зависит от заряда и напряжения между обкладками.

2. С = зависит от заряда и напряжения между обкладками.

3. С = зависит от размеров, формы и окружающей среды.

6. Определить заряд батареи конденсаторов, соединенных так, как показано на рис. 1. Емкость каждого конденсатора (в мкФ) указана на рисунке.

Рис.1.

Лабораторная работа №7

Определение электродвижущей силы и внутреннего сопротивления источника электрической энергии.

Контрольные вопросы.

1. Какова физическая суть электрического сопротивления?

Ответы: 1. Электрическое сопротивление характеризует противодействие электрическому току, обусловленное взаимодействием электронов с разного рода нарушениями кристаллического строения.

2. Электрическое сопротивление характеризует противодействие электрическому току, обусловленное столкновениями электронов с атомами и ионами в узлах кристаллической решетки.

2. Какова роль источника тока в электрической цепи?

Ответы: 1. Для создания свободных носителей зарядов в электрической цепи.

2. Для уменьшения сопротивления в замкнутой цепи.

3. Для поддержания стационарного электрического поля, чтобы существовал длительно в проводах электрический ток.

3. Каков физический смысл ЭДС? Дать определение вольту.

Ответы: 1. Зависимость электрической энергии, приобретенной зарядом во всей цепи, от устройства внешней цепи. 1В – напряжение в точке поля, при работе сил электрического поля равным 1 Дж.

2. Э.Д.С. характеризует зависимость электрической энергии, приобретенной зарядом в источнике тока от внутреннего устройства последнего. 1В – напряжение между двумя точками поля, когда поле совершает работу в 1 Дж при перемещении заряда 1 Кл между этими точками.

4. Соединить на короткое время вольтметр с источником элек­трической энергии, соблюдая полярность. Сравнить его показание с вычисленным по результатам опыта.

Ответы: 1. Будет равным Е. 2. Будет больше Е. 3. Будет меньше Е.

5. От чего зависит напряжение на зажимах источника тока?

Ответы: 1. От внешнего сопротивления R.

2. От внутреннего сопротивления r .

3. От внешнего R и внутреннего r сопротивлений.

4. Не зависит.

6. Пользуясь результатами произведенных измерений, опреде­лить напряжение на внешней цепи.

Ответы: 1. U = 2. U = 3. Е =

Лабораторная работа №8

Определение удельного сопротивления проводника.

Контрольные вопросы.

1. Почему удельное сопротивление проводника зависит от рода материала его?

Ответы: 1. Т.к. у разных металлов сосредоточена разная масса в единице объема (т.е. разная плотность).

2. Т.к. у разных металлов своя концентрация свободных электронов и расстояние между узлами кристаллической решетки.

2. Зависит ли удельное сопротивление от температуры?

Ответы: 1. Увеличивается. 2. Уменьшается. 3. Не изменяется.

3. Удельное сопротивление фехраля 1,1·10-6 Ом ·м. Что это значит? Где можно использовать такой материал?

Ответы: 1. Это значит, что 1м при сечении 1м2 фехралевый проводник имеет сопротивление 1,1·10-6Ом.

2. Это значит, что 1м любого сечения фехралевый проводник имеет сопротивление 1,1·10-6Ом.

4. Назвать известные вам методы определения сопротивления резистора?

Ответы: 1. С помощью амперметра и вольтметра.

2. С помощью омметра.

3. Зная какого рода проводник, с помощью весов и микрометра.

4. С помощью амперметра и весов.

5. С помощью вольтметра и штангенциркуля.

5. Как изменится напряжение на участке ОВ электрической це­пи (рис. 1), если медную проволоку на этом участке заменить ни­келиновой.

Ответы: 1. Увеличится.

2. Уменьшится.

3. Не изменится.

Рис.1

6. Определить сопротивление и длину медной проволоки массой 89 г, сечением 0,1 мм2. (Переведите все единицы измерения в систему СИ, дополнительные величины взять из таблицы в методичке).

Ответы: 1. 1,3 Ом. 2. 17 Ом. 3. 240 Ом.

Лабораторная работа №9

Изучение последовательного и параллельного соединения проводников.

Контрольные вопросы.

1. Восемь резисторов соединили по два последовательно в четыре параллельные ветви. Какая схема соответствует этим условиям.

Ответы: 1. Рис.1. 2. Рис.2.

Рис.1 Рис.2

2. Потребители электрической энергии соединены так, как показано на рис3. Определить эквивалентное сопротивление в этом случае, если R1= R2 =R3 =12 Ом, R4 = 4Ом, R5 = R6 = 40 Ом.

Ответы: 1. 80 Ом. 2. 28 Ом. 3. 36 Ом.

Рис.3

3. Учащийся при измерении напряжения на лампочке включил по ошибке амперметр вместо вольтметра. Что при этом произойдет?

Ответы: 1. Лампа не загорится, поскольку при таком включении почти все напряжение падает на вольтметре, у которого сопротивление больше, чем у лампы.

2. В цепи возникает очень большой ток (практически – короткое замыкание, т.к. R амперметра очень мало), а это ведет к порче амперметра (зашкаливание или перегорание катушки).

4. Изменится ли показание вольтметра рис.4 , если в участок, состоящий из нескольких параллельно соединенных резисторов, добавить еще один?

Ответы: 1. Нет.

2. Увеличится.

3. Уменьшится.

Рис. 4

5. Что изменилось на данном участке цепи, если включенный последовательно с ним амперметр показал увеличение силы тока?

Ответы: 1. Сопротивление увеличилось.

2. Сопротивление уменьшилось.

3. Сопротивление не изменилось.

6. Как включены 10 ламп для освещения трамвайного вагона, рассчитанных на напряжение 120 В? Напряжение в трамвайной сети 600 В.

Ответы: 1. Рис.1 2. Рис.2 3. Рис. 3 4. Рис.4

Рис.1 Рис.2 Рис.3 Рис.4

Лабораторная работа №10

Исследование зависимости мощности, потребляемой лампой накаливания, от напряжения на её зажимах.

Контрольные вопросы.

  1. Каков физический смысл напряжения на участке электриче­ской цепи?

Ответы: 1. Показывает, как быстро совершается работа электрическим током на участке электрической цепи.

2. Показывает тепловое действие тока.

3. Показывает изменение внутренней энергии на этом участке цепи.

2. Какие способы определения мощности тока вам известны?

Ответы: 1. Ваттметром. 4. Вольтметром и омметром.

2. Амперметром и вольтметром. 5. Счетчиком электрической энергии.

3. Амперметром и омметром. 6. Источником электрической энергии.

  1. Лампы, 200-ваттная и 60-ваттная, рассчитаны на одно напря­жение. Сопротивление какой лампы больше? Во сколько раз?

Ответы: 1. R1 > R2 в <~ 3,3 раза 2. R1 < R2 в <~ 3,3 раза 3. R1 = R2

4. Какое количество электроприборов одинаковой мощности (100 Вт) может быть включено в электрическую цепь напряжени­ем 220 В при номинальной силе тока в предохранителе (для этой цепи) 5 А?

Ответы: 1. 2. 20 3. 11

5. Какова причина укрупнения единичных мощностей энерго­блоков электростанций страны?

Ответы: 1. Чтобы попеременно использовать единичные мощности энергоблоков в отдельности.

  1. Более удобные технологические возможности для их изготовления.

  2. Меньший расход потребляемой энергии.

Лабораторная работа №11

Определение температурного коэффициента сопротивления меди.

Контрольные вопросы.

1. Какова физическая сущность электрического сопротив­ления?

Ответы: 1. Электрическое сопротивление характеризует противодействие электрическому току, обусловленное взаимодействием электронов с разного рода нарушениями кристаллического строения.

2. Электрическое сопротивление характеризует противодействие электрическому току, обусловленное кинетической энергией электронов, её изменением.

  1. Как объяснить увеличение сопротивления металлов при на­гревании?

Ответы: 1. При повышении температуры увеличивается число столкновений электронов с атомами и ионами в узлах кристаллической решетки.

2. При повышении температуры увеличивается число дефектов кристаллической решетки — выпавших из узлов атомов и оставшихся ими пустых мест.

3. Найти формулу, по которой определяется температур­ный коэффициент сопротивления.

Ответы: 1. 2. 3.

4. Почему температурный коэффициент сопротивления для электролитов отрицательный?

Ответы: 1. При повышении температуры сопротивление электролитов повышается.

2. При повышении температуры сопротивление электролитов понижается.

3. При повышении температуры сопротивление электролитов не изменяется.

5 . Каково сопротивление 0,5 кг медной проволоки диаметром 0,3 мм? (Переведите единицы измерения в систему СИ, дополнительные данные найдите в таблицах по методичке).

Ответы: 1. 20 Ом. 2. 186 Ом. 3. 26·103 Ом.

6. Указать практическое применение зависимости сопротивле­ния проводника от температуры.

Ответы: 1. Применяют в термометрах сопротивления с точностью до 0,00001 °С.

2. В автоматических устройствах.

3. В качестве эталонных сопротивлений в реостатах используют константан и манганин.

Лабораторная работа №12

Определение электрохимического эквивалента меди.

Контрольные вопросы.

1. Почему молекулы соли, кислоты и щелочи в воде распада­ются на ионы?

Ответы: 1. Происходит диссоциация и нагрев.

2. Происходит ионизация.

3. Происходит термоэлектронная эмиссия.

2. Почему с повышением температуры сопротивление электро­лита уменьшается?

Ответы: 1. Увеличивается количество носителей заряда в электролите, т.е. увеличивается степень диссоциации.

2. Потому что происходит все больше объединение ионов разных знаков в нейтральные молекулы.

3. Электроны увеличивают свою скорость.

3. Будет ли происходить электролитическая диссоциация в ус­ловиях космического полета?

Ответы: 1. Да. 2. Нет.

4. При каких условиях концентрация электролита в процессе электролиза остается постоянной? Меняется?

Ответы: 1. При электролизе, сопровождающемся растворением анода, концентрация электролита меняется во всех случаях.

2. При электролизе, сопровождающемся растворением анода, концентрация электролита остается постоянной, во всех остальных случаях электролиза меняется.

5. Как следует поступить, если по ошибке при выполнении опы­та взвешенная пластинка была соединена с положительным полю­сом источника тока?

Ответы: 1. Повторить опыт, поменяв знаки электродов.

2. Взвесить эту пластину (растворяющийся анод) и уменьшенную массу использовать.

3. Взвесить другой отрицательный электрод (катод).

6. Как поступают, когда необходимо к угольному электроду припаять провод?

Ответы: 1. Зажимают в специальную трубку или закрепляют хомутками металлическими.

2. Непосредственно припаивают к электроду.

3. Просверливают отверстия, вставляют металлические штыри.

Лабораторная работа №13.

Снятие вольтамперной характеристики полупроводникового диода.

Контрольные вопросы.

1. В чем различие проводимости проводников и полупроводни­ков?

Ответы: 1. Проводимость у проводников создается электронами и дырками, а у полупроводников только электронами.

2. Проводимость у проводников создается электронами, а у полупроводников электронами и дырками.

2. Как объяснить уменьшение удельного сопротивления полупроводника при уменьшении температуры?

Ответы: 1. При нагревании электроны увеличивают свою скорость, образуя диполи.

2. Разрывается ковалентная связь и образуется большое количество свободных носителей заряда.

3. Что является в схеме триода на рис. 1. входной цепью и что — выходной?

Ответы: 1. и

2. — выходное и — входное

3. и — входное

Рис.1.

4. Как следует включить в цепь транзистор, чтобы он действовал и как диод в прямом направлении?

Ответы: 1. Рис.2 2. Рис.3 3. Рис.4

. Рис.2 Рис.3 Рис.4

5. Что показывает вольтамперная характеристика диода?

Ответы: 1. Зависимость силы тока от напряжения в прямом (пропускном) и обратном (запирающем) направлении I = f (U).

2. Зависимость сопротивления диода от температуры R = f (t).

3. Зависимость напряжения от температуры U = f(t).

6. На рис.5 изображена вольтамперная характеристика полупроводникового диода. Определить, чему равно внутреннее сопротивление диода в пропускном направ­лении при U = 0,3 В, в запирающем направлении при U = —400 В. Объяснить, почему сила тока в проходном направлении с увели­чением напряжения растет очень быстро, в запирающем же направлении — очень мала и почти не меняется с ростом на­пряжения.

Ответы: 1. 0,15 Ом — в пропускном

0,08 Ом — в запирающем

2. 167 Ом — в пропускном

5·10-7 Ом — в запирающем

3. 6 Ом — в пропускном

2·106 Ом — в запирающем

Нахождение емкости конденсатора. Наука техника технологии

Плоский конденсатор состоит из двух параллельных пластин, разделённых небольшим зазором шириной , заполненным однородным диэлектриком.

Нам известно, что поле между двумя разноимённо заряженными пластинами с одинаковой по величине поверхностной плотностью равно, где,S– площадь каждой пластины. Напряжение между обкладками:

Используя определение емкости конденсатора, получаем:

Отметим, что полученная формула является приближенной, так как выведена без учета искажения поля у краев пластин. Расчет по этой формуле дает завышенное значение ёмкости и тем точнее, чем меньше зазор по сравнению с линейными размерами пластин.

Ёмкость сферического конденсатора.

Сферический конденсатор представляет собой систему двух концентрических сфер с радиусами и. Электрическое поле между обкладками сферического конденсатора согласно теореме Гаусса определяется зарядом внутренней сферы. Напряжение между обкладками равно:

.

Для ёмкости сферического конденсатора получаем:

Это формула точная.

Если , полученная формула переходит в выражение для ёмкости плоского конденсатора.

Ёмкость цилиндрического конденсатора.

Цилиндрический конденсатор составляет систему двух коаксиальных цилиндров с радиусами и, длиной.

Рассуждая аналогично выводу ёмкости сферического конденсатора, получаем:

..

Полученная формула является приближенной и при малом зазоре переходит в формулу емкости плоского конденсатора.

Соединение конденсаторов.

В практике для получения необходимых значений емкости используют соединения конденсаторов: а) последовательное, б) параллельное, в) смешанное (см. рисунок).


Ёмкость последовательного соединения конденсаторов.

Заряды последовательно соединенных конденсаторов равны , а напряжение на батарее. Из определения емкости следует:

Если , то(ёмкость последовательного соединения меньше наименьшей ёмкости в последовательном соединении).

Для последовательно соединенных конденсаторов емкость вычисляется по формуле:

Ёмкость параллельного соединения конденсаторов.

Заряд батареи равен сумме зарядов:

а напряжение . По определению емкости получаем:

Для параллельно соединенных конденсаторов:.

В случае одинаковых конденсаторов: .

Оценить емкость батареи (см. рисунок) .

Используя свойство бесконечности можно представить цепь в виде соединения (см. рисунок).

Для расчета ёмкости батареи получаем:

Откуда: , так как, то.

Лекция 7.

Диэлектрики в электрическом поле.

Диэлектриками (изоляторами) называют вещества, не проводящие постоянного электрического тока. Это означает, что в диэлектриках отсутствуют «свободные» заряды, способные перемещаться на значительные расстояния.

Диэлектрики состоят либо из нейтральных молекул, либо из ионов, находящихся в узлах кристаллической решетки. Сами же молекулы могут быть полярными инеполярными. Полярные молекулы обладают дипольным моментом, у неполярных молекул дипольный момент равен нулю.

Поляризация.

В электрическом поле диэлектрики поляризуются. Это явление связано с появлением в объеме и на поверхности диэлектрика «связанных » зарядов. При этом конечный объем диэлектрика приобретает дипольный момент. Механизм поляризации связан с конкретным строением диэлектрика. Если диэлектрик состоит из неполярных молекул, то в пределах каждой молекулы происходит смещение зарядов – положительных по полю, отрицательных против поля, т.е. молекулы, приобретают дипольный момент. У диэлектрика с полярными молекулами в отсутствии внешнего электрического поля их дипольные моменты ориентированы хаотично.

Под действием электрического поля диполи ориентируются преимущественно в направлении поля. Рассмотрим подробнее этот механизм (см. рисунок). Пара сил исоздает вращательный момент равный, где- дипольный момент молекулы. Этот момент стремится ориентировать диполь вдоль поля. В ионных кристаллах под действием электрического поля все положительные ионы смещаются по полю, отрицательные – против поля. Отметим, что смещение зарядов очень малы даже по сравнению с размерами молекул. Это связано с тем, что напряженность внешнего электрического поля обычно много меньше напряженности внутренних электрических полей в молекулах.

Отметим, что существуют диэлектрики, поляризованные даже при отсутствии внешнего поля (электреты, сегнетоэлектрики). Мы остановимся на рассмотрении только однородных диэлектриков, в которых отсутствует остаточная поляризация, а объемный и «связанный» заряд всегда равен нулю .

Плоский конденсатор – это физическое упрощение, взявшее начало из ранних исследований электричества, представляющее собой конструкцию, где обкладки имеют форму плоскостей и в каждой точке параллельны.

Формулы

Многие ищут формулы, описывающие ёмкость плоского конденсатора. Если это так, то не читайте ниже любопытные и малоизвестные факты, потому что сухие математические знаки, конечно же, важнее.

Первым определил ёмкость плоского конденсатора Вольта. В его распоряжении ещё не было такой величины, как разница потенциалов, именуемая напряжением, но интуитивно он совершенно правильно объяснил суть явления. Что касается количества зарядов, то он трактовал её, как объем электрического флюида атмосферы – не совсем правильно, но в конечном итоге похоже на правду. Согласно этому мировоззрению ёмкость плоского конденсатора может быть найдена, как отношение объёма накопленного электрического флюида к разнице атмосферных потенциалов, то есть:

Эта формула применима к любому конденсатору, вне зависимости от его конструкции. То есть, является универсальной. Специально для плоских конденсаторов имеется формула ёмкости, выраженная через свойства материала диэлектрика и геометрические размеры:

В этой формуле через S обозначена площадь обкладок, вычисляемая через произведение сторон, а d – показывает расстояние между обкладками. Прочие символы – электрическая постоянная (8,854 пФ/м) и диэлектрическая проницаемость материала диэлектрика, да простит Тот столь откровенные тавтологии. Электролитические конденсаторы обладают столь большой ёмкостью по той причине, что проводящий раствор отделен от металла очень тонким слоем оксида. Следовательно, d в этом случает будет минимальным. Единственный минус в том, что электролитические конденсаторы полярные, их нельзя подключать в цепи переменного тока. С этой целью на каждом анод или катод обозначены значками плюса или минуса.

Плоские конденсаторы сегодня редко встречаются, и это преимущественно плёночные микроскопические технологии, где такой род поверхностей является доминирующим. Все пассивные и активные элементы образуются через трафарет. И, следовательно, имеют вид плёнок. Плоские индуктивности, резисторы и конденсаторы наносятся в виде токопроводящих паст.

От материала диэлектрика ёмкость зависит по той причине, что у каждого из них структура своя. Считается, что аморфное вещество состоит из неориентированных диполей, упруго укреплённых на своих местах. При приложении внешнего электрического поля они обратимо ориентируются вдоль силовых линий, ослабляя напряжённость. В результате заряд продолжает накапливаться, пока этот процесс не прекратится. По мере выхода энергии из обкладок диполи возвращаются на свои места, делая возможным следующий рабочий цикл. Так функционирует плоский электрический конденсатор.


Из истории

Исторически первым начал исследовать накопление заряда великий Алессандро Вольта. В докладе Королевскому научному обществу за 1782 год он впервые озвучил слово конденсатор. В понимании Вольты электрофорус, представляющий собой две параллельные обкладки, выкачивал из эфира электрический флюид.

В то время все познания сводились к тому, что учёные думали, будто атмосфера Земли содержит в себе нечто, что не может быть определено приборами. Существовали только простейшие электроскопы, способные определить знак заряда и его наличие, но не дававшие представления о количестве. Учёные просто натирали мехом поверхность тела и подносили его для исследования в область влияния прибора. Ещё Гильберт показал, что электрические и магнитные взаимодействия ослабевают с расстоянием. Поэтому учёные примерно знали, что нужно делать, но исследования не продвигались ни на йоту.

Гипотеза об атмосферном электричестве высказана Бенджамином Франклином. Он активно исследовал молнии и пришёл к выводу, что это проявления все той же единой силы. Запуская воздушного змея в небо, он соединял его шёлковой нитью с землёй и наблюдал дуговой разряд. Это были достаточно опасные опыты, и Бенджамин много раз рисковал своей жизнью ради развития науки. О том, что шёлковая нить проводит статический заряд, было известно от Стивена Грея, первым собравшего в 1732 году электрическую цепь.

Уже через 20 лет (1752 год) Бенджамин Франклин предложил конструкцию первого громоотвода, осуществлявшего молниезащиты близлежащих построек. Только вдуматься! – до этого всякий мог ожидать того, что его дом сгорит от случайного удара. Именно Бенджамин Франклин предложил один из видов заряда называть положительным (стеклянный), а другой отрицательным (смоляной). Так физики были введены в заблуждение относительно истинного направления движения электронов. Но как они могли думать иначе, когда в 1802 году на примере опытов нашего соотечественника Петрова увидели, что на аноде образуется ямка? Следовательно, положительные частицы переносили заряд на катод, вот только это были ионы воздушной плазмы.

К началу исследования Вольтой электрических явлений, таким образом, были уже известны статические заряды и факт наличия у них двух знаков, кроме того люди упорно считали, что весь «флюид» берётся из воздуха. На эту мысль их натолкнули опыты с натиранием янтаря шерстью, которые не могли быть проведены под водой. Следовательно, логичным было предположить, что электричество может происходить только из атмосферы Земли, что, конечно же, совершенно неверно. В частности, многие растворы, исследованные Хампфри Дэви, могут проводить электрический ток.

Причина, следовательно, была в другом – при натирании янтаря под водой силы трения снижались в десятки и сотни раз, а заряд рассеивался по всему объёму жидкости. Следовательно, этот процесс был всего лишь неэффективным. Но сегодня каждый добытчик знает, что нефть прекрасно электризуется трением о трубы и без воздуха. Следовательно, атмосфера для «флюида» не является обязательным компонентом.

Самый большой в мире плоский конденсатор

Столь систематизированные, но в корне неверные толкования все-таки не смогли остановить Вольту на его исследовательском пути. Он упорно изучал электрофорус, как один из самых совершенных генераторов, существовавших в то время. Вторым был серный шар Отто фон Герике, изобретённый более чем за век до этого (1663 год). С тех самых пор его конструкция мало менялась, но после открытий Стивена Грея заряд начали снимать при помощи проводников. В частности, в для этого служат металлические гребёнки-нейтрализаторы.

Долгое время учёные ходили вокруг да около. Электрофорная машина 1880 года может считаться первым мощным генератором разряда, позволявшим получить дугу, но своей настоящей силы электроны достигли в генераторе Ван де Граафа (1929 год), где разница потенциалов составила единицы мегавольта. Для сравнения грозовое облако, согласно данным Википедии, может иметь потенциал относительно Земли в единицы гигавольт (на три порядка больше, чем в человеческой машине).

Суммируя сказанное, можно с определённой долей уверенности сказать, что природные процессы используют в качестве принципа своего действия электризацию трением, влиянием и некоторые другие её виды, а мощный циклон является самым большим из известных нам плоских конденсаторов. Молния показывает, что бывает, когда диэлектрик (атмосфера) не выдерживает приложенной разницы потенциалов и пробивается. В точности то же самое происходит в любом плоском конденсаторе, созданном человеком, если вольтаж оказывается для него непомерным. Пробой твёрдого диэлектрика необратим, а возникающая электрическая дуга часто служит причиной расплавления обкладок и выхода изделия из строя.

Электрофорус

Итак, Вольта взялся за исследование модели природных процессов. Первый электрофорус появился в 1762 году сконструированный Йоханом Карлом Вильке. По-настоящему популярным прибор становится после докладов Вольты Королевскому научному обществу (середина 70-х годов XVIII века). Вольта же и дал прибору его нынешнее название.


Электрофорус способен накапливать электростатический заряд, образованный трением резины куском шерсти. Он состоит из двух плоских, параллельных друг другу обкладок:

  • Нижняя представляет собой тонкий кусок резины. Толщина его выбирается из соображений эффективности устройства. Если выбрать кусок более солидный, то значительная часть энергии будет накапливаться внутри диэлектрика на ориентацию его молекул. Что и наблюдается в современном плоском конденсаторе, куда диэлектрик помещается для увеличения электроёмкости.
  • Верхняя пластина из тонкой стали кладётся сверху, когда заряд уже накоплен трением. За счёт влияния на верхней поверхности образуется избыток отрицательного заряда, и он должен быть снят на заземлитель, чтобы при расстыковке двух обкладок не произошло взаимной компенсации.

Принцип действия этого плоского конденсатора должен быть уже понятен. Оператор трёт резину шерстью, оставляя на ней отрицательный заряд. Затем сверху кладётся кусок металла. Из-за значительной шероховатости поверхностей они не соприкасаются, но находятся на некотором расстоянии друг от друга. В результате металл электризуется влиянием. Электроны отталкиваются поверхностным зарядом резины и уходят на внешнюю плоскость, где оператор их снимает через заземлитель лёгким кратковременным прикосновением.

Низ металлической обкладки остаётся заряженным положительно. При расстыковке двух поверхностей этот эффект сохраняется, потому что в материале наблюдается дефицит электронов. И можно наблюдать искру, если дотронуться до металлической обкладки. Этот опыт можно на одном и том же заряде резины проделывать сотни раз, поскольку её поверхностное статическое сопротивление весьма велико. Это не даёт заряду растекаться. Демонстрируя этот опыт, Вольта привлёк внимание всего научного мира, но исследования никак не двигались вперёд, если не считать открытий Шарля Кулона.

В 1800 году сам Алессандро даёт толчок развитию изысканий в области электричества, изобретя свой знаменитый гальванический источник питания.

Конструкция плоского конденсатора

Электрофорус по сути представляет собой первый из когда-либо сконструированных плоских конденсаторов. Его обкладки способны хранить только статический заряд, потому что иначе наэлектризовать резину невозможно. Поверхность очень долго хранит электроны. Вольта даже предлагал снимать их пламенем свечи через ионизированный воздух или ультрафиолетовым излучением Солнца. Сегодня каждый школьник знает, что то же самое можно проделать и водой. Правда, электрофорус нужно будет после этого высушить.

В современном мире нижней обкладкой может служить тефлоновое покрытие или пластик. Они тоже хорошо набирают статический заряд. Диэлектриком здесь служит воздух. Чтобы перейти к конструкции современного конденсатора, нужно обе обкладки сделать металлическими. Тогда при возникновении на одной из них заряда влиянием электризация распространится на вторую, и если другой контакт заземлён, накопленная энергия может храниться какое-то время.


Запас электронов напрямую зависит от материала диэлектриков. Так например, среди современных конденсаторов встречаются:

  1. Слюдяные.
  2. Воздушные.
  3. Электролитические (оксидные).
  4. Керамические.

В эти названия как раз и заложен материал диэлектрика. От его состава зависит напрямую ёмкость, которая может быть увеличена во много раз. Роль диэлектриков объяснялась выше, в частности их параметры определяются непосредственно строением вещества. Однако многие материалы, обладающие высокими характеристиками, использовать не удаётся по причине их непригодности. Так например, вода обладает высокой диэлектрической проницаемостью.

КОНДЕНСАТОР — означает накопитель. В радио и электронной аппаратуре конденсатор является накопителем электрических зарядов. Простейший конденсатор состоит из двух металлических пластинок разделенных слоем диэлектрика. Диэлектрик — это материал который не проводит электрического тока и обладает определенными свойствами о которых поговорим чуть позже.

Так как конденсатор является накопителем, то он должен обладать определенной емкостью (объемом для накопления зарядов). На емкость конденсатора влияют площадь пластин (еще их называют «обкладками»), расстояние между обкладками и качество диэлектрика. К хорошим диэлектрикам относятся вакуум, эбонит, фарфор, слюда, полиэтилен, текстолит и много других синтетических материалов.
На рисунке изображен простейший конденсатор с двумя параллельными обкладками площадью S (S = m * n), которые находятся в вакууме на расстоянии d друг от друга.


Если между верхней и нижней обкладками конденсатора приложить напряжение Uab, то на верхней и нижней обкладках конденсатора накопятся одинаковые положительный +q и отрицательный -q заряды, которые называют свободными. Между обкладками возникает электрическое поле обозначенное на рисунке буквой Е.
Емкость нашего конденсатора (обозначается буквой С) будет: С = Eo*S/d, где Ео — электрическая постоянная (для вакуума) Ео=8,854 * 10 -12 Ф/м (Фарад на метр).
Если между обкладками поместить диэлектрик,


то ёмкость конденсатора будет: С = Er * Eo *S / d. В формуле расчета ёмкости добавилась величина Er — относительная диэлектрическая проницаемость введённого диэлектрика.
Из формулы следует, что емкость конденсатора увеличивается на величину Er проницаемости диэлектрика. Итак, чем больше площадь S пластин конденсатора, больше значение Er и меньше расстояние d между пластинами, тем больше емкость конденсатора. Основной единицей емкости в системе единиц СИ является фарад (Ф). Емкость 1Ф очень велика. В электротехнике обычно используют дольные единицы емкости:
микрофарада (мкФ), 1мкФ = 1*10 -6 Ф,
нанофарада (нФ), 1нФ = 1*10 -9 Ф, и
пикофарада (пФ), 1пФ = 1*10 -12 Ф.




При выборе диэлектрика для конденсаторов, кроме относительной диэлектрической проницаемости диэлектрика, учитывают еще два важных параметра:
1) Электрическую прочность — прочность диэлектрика при подаче на прокладки конденсатора высокого напряжения. При низкой электрической прочности может произойти электрический пробой, и диэлектрик станет проводником электрического тока;
2) Удельное объемное сопротивление — электрическое сопротивление диэлектрика постоянному току. Чем больше удельное сопротивление диэлектрика, тем меньше утечка накопленных зарядов в конденсаторе.

КОНДЕНСАТОР В ЦЕПИ ПОСТОЯННОГО ТОКА. На графике накопление заряда конденсатором выглядит как показано на рисунке 1.


Время заряда конденсатора зависит от ёмкости конденсатора (при одинаковом приложенном напряжении). Чем больше ёмкость конденсатора, тем больше время заряда. Аналогичная картина (Рис. 2) наблюдается при разрядке конденсатора на сопротивление. При одинаковом сопротивлении время разряда больше у конденсатора с большей ёмкостью.

КОНДЕНСАТОР В ЦЕПИ ПЕРЕМЕННОГО ТОКА. Если напряжение приложенное к емкостному элементу, будет изменяться по амплитуде (переменное напряжение),то будет изменяться и заряд конденсатора, то есть в емкостном элементе появится ток.



Ток Ic проходящий через конденсатор зависит от частоты f приложенного переменного напряжения и ёмкости С конденсатора. Если для постоянного тока сопротивление конденсатора можно считать равным бесконечности, то для переменного тока конденсатор обладает определённым сопротивлением. Сопротивление конденсатора переменному току Rc рассчитывается по формуле показанной на рисунке.
В формуле расчета емкостного сопротивления переменному току частота выражается в герцах, а емкость конденсатора в фарадах. Из формулы видно, что с увеличением частоты f при неизменной емкости конденсатора сопротивление Rc снижается, аналогично с увеличением емкости конденсатора при неизменной частоте сопротивление Rc так же снижается. Конденсаторы, так же как и резисторы, для получения заданной емкости Со можно включать параллельно и последовательно. Формулы расчета результирующей емкости показаны на рисунке.



КОНСТРУКЦИЯ, ПАРАМЕТРЫ И ТИПЫ КОНДЕНСАТОРОВ. Предположим, что мы конструируем конденсатор и попробуем, уже обладая определенными знаниями, рассчитать емкость конденсатора. Как известно, емкость конденсатора зависит от площади обкладок S, расстояния между обкладками d и диэлектрической проницаемости применяемого диэлектрика Er. Обкладки конденсатора изготавливаются из металлов с хорошей электрической проводимостью — алюминий, медь, серебро, золото. Емкость конденсатора не зависит от толщины обкладок, поэтому чем тоньше обкладки конденсатора, тем лучше — экономим металл и уменьшаем геометрический объём конденсатора.


Расстояние d не должно быть слишком малым, во избежание электрического пробоя диэлектрика.
Выберем в качестве диэлектрика наиболее распространенный материал — гетинакс с Er равной 6 … 8. Примем Er для нашего конденсатора равной 7.


Площадь S вычисляется для одной обкладки конденсатора при условии, что линейные размеры обкладок одинаковы. Если одна из обкладок имеет меньшие длину или ширину то площадь вычисляется для меньшей обкладки.
Все размеры — длина и ширина обкладок и расстояние между ними должны быть выражены в метрах. Примем размеры такие, какие показаны на рисунке. Подставим в формулу расчета емкости конденсатора наши данные: C = Er * Eo * S / d;
C = 7 * 8.854*10 -12 * 0.0025 / 0.001= 0.000000000155Ф (фарады).
Возведем полученный результат в 12 степень чтобы получить значение емкости в пикофарадах:
C = 0.000000000155 12 = 155пФ.
Полученная нами ёмкость конденсатора 155пф очень мала, обычно такие ёмкости используются в аппаратуре работающей на высоких частотах переменного тока порядка 1 — 600 МГц (мегагерц).
Представьте себе, что мы разрабатываем миниатюрный карманный радиоприемник в котором требуется порядка 30 таких конденсаторов.

Если мы установим в схему 30 разработанных нами конденсаторов, не считая других необходимых радиодеталей, то наш радиоприемник никак не получится миниатюрным. Все дело в том, что объём только наших конденсаторов получится таким, что его никак нельзя будет назвать приемлемым.
Объем одного конденсатора Vc равен Vc = 5см * 5см * 0,1см
Vc = 2,5см в кубе. Тогда объем 30 конденсаторов будет равен:
V = 30 * 2,5 = 75см в кубе.
Что делать, как быть, как уменьшить геометрический объем конденсатора для применения в миниатюрной радиоаппаратуре? Для решения этой проблемы максимально уменьшают расстояние между обкладками, тогда увеличивается емкость и уменьшается геометрический объем конденсатора. Но расстояние уменьшают до определенных пределов иначе конденсатор будет пробиваться даже при низком напряжении подаваемом на конденсатор. В связи с этим на каждом конденсаторе указывается напряжение которое он может выдержать.

Для уменьшения площади обкладок конденсатор делают многослойным состоящим как бы из нескольких параллельно включенных конденсаторов (вспомните формулу параллельного включения конденсаторов).
В качестве диэлектрика в миниатюрных конденсаторах используют тонкие пленки из синтетических материалов, а в качестве обкладок металлическую фольгу, чаще всего из алюминия.



На корпусе конденсатора, обычно, указывается его тип, емкость и рабочее напряжение. Остальные параметры конденсатора определяются из справочников. Емкость конденсатора указывается не так, как на электрических схемах. Например емкость 2,2пФ обозначается 2П2, емкость 1500 пФ — 1Н5, емкость 0,1 мкФ — М1, емкость 2,2 мкФ — 2М2, емкость 10 мкФ — 10М.
У обычных конденсаторов КМ, КД, МБМ и так далее трудно получить большую ёмкость при малых габаритах поэтому были разработаны так называемые электролитические конденсаторы у которых в качестве диэлектрика используется специальная электролитическая жидкость с очень большим Er. Ёмкость таких конденсаторов может достигать сотен тысяч микрофарад. К недостатку таких конденсаторов следует отнести низкое рабочее напряжение (до 500V) и обязательное соблюдение полярности при включении в схему.
Для настройки и подстройки некоторых типов радиоаппаратуры, например радиоприемник или телевизор, применяют специальные конденсаторы с изменяемой ёмкостью.

В зависимости от назначения такие конденсаторы называют «подстроечные» и «конденсаторы переменной емкости».
Емкость переменных и подстроечных конденсаторов изменяется механическим способом, путем изменения расстояния между обкладками или изменения площади пластин. В качестве диэлектрика в таких конденсаторах используется воздух или фарфор.
В заключение следует отметить, что в настоящее время, в связи с бурным развитием радиоэлектроники подстроечные и переменные конденсаторы практически не применяются. Их с успехом заменяют специальные фильтры и полупроводниковые приборы которые не требуют механического изменения параметров.

Емкость

и конденсатор

Устройство, обеспечивающее чистую емкость, называется конденсатором. Конденсатор — это двухконтактный накопитель энергии, который накапливает энергию в своем электрическом поле. Встречается в цепях поворота и фильтрации.

Он состоит из двух металлических пластин, обращенных друг к другу и разделенных диэлектрической средой, такой как воздух, керамика, слюда или бумага. Эти пластины могут нести в себе электрические заряды.

Имея две пластины очень близко друг к другу и разделенные диэлектрической средой, можно собирать большое количество заряда на пластинах.Когда на пластины подается напряжение, источник создает положительный заряд на одной пластине и отрицательный заряд на другой пластине. Заряды видят противоположный заряд на противоположной пластине и притягиваются к нему, а не противостоят аналогичным зарядам на той же пластине.

Определение емкость

Емкость можно определить как свойство конденсатора, которое позволяет конденсатору накапливать заряды на своих проводящих пластинах. Это зависит от физических размеров пластин и проницаемости разделяющего их диэлектрика.

Емкость конденсатора изменяется, как показано ниже:

  • Чем больше площадь пластин, тем выше будет емкость.
  • Больше расстояние разделения между пластин, меньше будет емкость.
  • Повышенная проницаемость разделительной средний, больше будет емкость.

Количество заряда q, хранящегося в конденсаторе, когда напряжение V, приложенный к клеммам конденсатора, равен

.

Q = CV

Где C — емкость конденсатора, а это отношение заряда одной пластины конденсатора к напряжению, приложенному между тарелки.

Единица емкости

Единица измерения емкости — фарад (Ф). Один фарад можно определить как меру емкости конденсатора, который может накапливать один кулон заряда на своих пластинах, когда к нему приложена разность потенциалов в один вольт. Единица конденсатора была названа в честь Мишеля Фарадея за открытие электромагнитной индукции, на основе которой работают двигатели и генераторы.

Соотношение напряжение-ток в конденсаторе

Как мы знаем, ток — это поток электронов.Дифференцируя приведенное выше уравнение ( Q = CV ) по времени t,

dQ / dt = C. dV / dt

dQ / dt — это скорость изменения сборов, которая есть не что иное, как Текущий. Следовательно,

Ток i = C. dV / dt

Следовательно, ток, протекающий через конденсатор, пропорционален скорости изменения напряжения на выводах конденсатора, и конденсатор предотвращает быстрое изменение этого напряжения. Но ток может течь мгновенно. Вот почему резистор предварительной зарядки используется перед конденсатором.Идеальный конденсатор не рассеивает энергию, а накапливает ее и полностью доставляет в схему.

На коммерческом рынке доступны различные типы конденсаторов. Самыми основными категориями являются конденсатор постоянной емкости, поляризованный конденсатор и конденсатор переменной емкости. Клеммы конденсатора постоянной емкости взаимозаменяемы. У поляризованных конденсаторов заранее определены анод и катод. При замене они могут привести к взрыву.

Конденсатор действует как разомкнутый переключатель в цепи постоянного тока.Когда на конденсатор подается постоянное напряжение, на обеих пластинах на короткое время накапливаются заряды. После накопления зарядов через него не проходит ток, и он действует как разомкнутый выключатель.

ОБЪЕМ

ЕМКОСТЬ ЕМКОСТЬ

Еще одно важное свойство цепей переменного тока, помимо сопротивления и индуктивности, это емкость. В то время как индуктивность представлена ​​в цепи катушкой, емкость представлена ​​конденсатором.

Любые два проводника, разделенные непроводником, называемым диэлектриком, составляют конденсатор. В электрической цепи конденсатор выполняет роль резервуар или хранилище для электричества.

Когда конденсатор подключен к источнику прямого ток, такой как аккумуляторная батарея в схеме, показанной на рисунке 8-176, и переключатель затем замыкается, пластина с маркировкой B становится положительно заряженной, и пластина А заряжена отрицательно.Ток течет во внешней цепи в течение времени электроны движутся от B к A. Текущий поток в цепи максимальна в момент замыкания переключателя, но постоянно затем уменьшается, пока не достигнет нуля. Ток становится равным нулю, когда как только разница в напряжении A и B станет такой же, как напряжение батареи. Если выключатель разомкнут, пластины остаются заряженными. Тем не мение, конденсатор быстро разряжается при коротком замыкании.

Количество электричества, которое может хранить конденсатор, зависит от нескольких факторов, включая тип материала диэлектрика. Прямо пропорционально площади пластины и обратно пропорционально расстоянию между тарелки.

На рисунке 8-177 размещены две плоские металлические пластины. близко друг к другу (но не касаясь). Обычно пластины электрически нейтральный; то есть ни на одной из пластин не будет электрического заряда.В момент замыкания переключателя в положение батареи счетчик покажет определенный скачок тока в одном направлении, но почти мгновенно вернется к нулю.

Если затем вынуть аккумулятор из цепи и выключатель замкнется в положении конденсатора счетчик снова показывает кратковременный скачок тока, но на этот раз в противоположном направлении. Из этого эксперимента очевидно что две пластины накапливают энергию при подключении к источнику напряжения, и высвобождает энергию при коротком замыкании.Две пластины составляют простой электрический конденсатор, или конденсатор, и обладают свойством хранения электричество. На самом деле энергия хранится в электрическом или диэлектрическом, поле между пластинами.

Также должно быть ясно, что во время работы конденсатора заряжен или разряжен, в цепи есть ток, даже если цепь разорвана зазором между пластинами конденсатора. Однако там ток только во время заряда и разряда, и в этот период времени очень мало.Не может быть постоянного движения постоянного тока через конденсатор. Хороший конденсатор блокирует постоянный ток (не пульсирует постоянного тока) и будет пропускать воздействие переменного тока.

Заряд электричества, который может быть помещен на конденсатор, пропорционален приложенному напряжению и емкости конденсатора (конденсатора). Емкость зависит от общей площади пластин, толщины диэлектрик и состав диэлектрика.

Если тонкий лист бакелита (заполненный слюдой) заменить воздух между обкладки конденсатора, например, емкость будет увеличена примерно пять раз.

Любой электрический заряд, произведенный приложенным напряжением и ограниченный изолятор (диэлектрик) создает диэлектрическое поле. Как только поле создан, он имеет тенденцию противодействовать любому изменению напряжения, которое может повлиять на его исходный должность. Все цепи содержат некоторую емкость, но если они не содержат единица, называемая конденсатором, емкость, для всех практических целей, игнорируется.Два проводника, называемые электродами или пластинами, разделены из непроводника (диэлектрика) составляют простой конденсатор. Пластины могут быть изготовленным из меди, олова или алюминия. Часто их делают из фольги. (металлы, спрессованные в тонкие листы и пригодные для прокатки). Диэлектрик может быть воздух, стекло, слюда или электролит, состоящий из оксидной пленки, но используемый тип будет определять величину напряжения, которое может быть приложено, и количество энергии, которое будет сохранено. Диэлектрические материалы имеют различные атомные структуры и представляют различное количество атомов для электростатическое поле.Все диэлектрические материалы сравнивают с вакуумом. и имеют числовое значение в соответствии с соотношением пропускной способности между их. Номер, присвоенный материалу, основан на той же площади и толщине. как используется в вакууме. Числа, используемые для выражения этого отношения, называются диэлектрические постоянные и обозначаются буквой «К.»

Если источник переменного тока заменен батареи, конденсатор действует совершенно иначе, чем с прямым Текущий.Когда в цепь подается переменный ток (рисунок 8-179), заряд на пластинах постоянно меняется. Это означает, что электричество сначала должен течь от Y по часовой стрелке до X, затем от X против часовой стрелки по кругу до Y, затем от Y по часовой стрелке до X и так далее. Хотя нет через изолятор между пластинами конденсатора протекает ток, он постоянно течет в оставшейся части цепи между X и Y. цепь, в которой есть только ёмкость, ток уводит втянутый напряжение по сравнению с цепью, в которой есть индуктивность, где ток отстает от напряжения.

Единицей измерения емкости является фарада, для которой обозначен символ буква «f». Фарад слишком велик для практического использования, и единицы измерения обычно используются микрофарады, одна миллионная фарада и микромикрофарада, одна миллионная микрофарада.

Типы конденсаторов

Конденсаторы можно разделить на две группы: фиксированные и переменные. Фиксированный конденсаторы, которые имеют приблизительно постоянную емкость, тогда могут быть далее делятся, в зависимости от типа используемого диэлектрика, на следующие классы: бумага, масло, слюда, электролитические конденсаторы.Керамические конденсаторы также используются в некоторых схемах.

При подключении электролитических конденсаторов в цепь соблюдайте полярность. необходимо соблюдать. Бумажные конденсаторы могут иметь одну клемму с пометкой «земля», Это означает, что этот вывод подключается к внешней фольге. Полярность делает обычно не требуется при использовании соединительной бумаги, масла, слюды или керамики. конденсаторы.

Бумажные конденсаторы

Пластины бумажных конденсаторов представляют собой полосы металлической фольги, разделенные вощеная бумага (рисунок 8-180).Диапазон емкости бумажных конденсаторов от примерно 200 ммс до нескольких мф. Полоски фольги и бумаги свернуты вместе, чтобы сформировать цилиндрический картридж, который затем запечатывается воском для не допускать попадания влаги и предотвращения коррозии и утечки Два металлических провода припаяны к пластинам, по одной на каждом конце цилиндра. Сборка заключена либо в картонную обложку, либо в твердый формованный пластиковое покрытие.

Конденсаторы ванны состоят из бумажных конденсаторных картриджей герметично закрывается в металлических контейнерах.Контейнер часто служит общая клемма для нескольких закрытых конденсаторов, но если это не клемма, крышка служит экраном от электрических помех (рисунок 8-181).
Масляные конденсаторы

В радио- и радиолокационных передатчиках напряжение достаточно высокое, чтобы вызвать искрение, или пробой бумажных диэлектриков. Следовательно, в конденсаторы, в которых используется масло или пропитанная маслом бумага для диэлектрический материал является предпочтительным.Конденсаторы этого типа значительно дороже обычных бумажных конденсаторов, и их использование обычно ограничивается радио- и радиолокационным передающим оборудованием (рисунок 8-182).

Слюдяные конденсаторы

Фиксированный слюдяной конденсатор изготовлен из металлических пластин из фольги, которые разделены между собой. листами слюды, образующими диэлектрик. Вся сборка покрыта из литого пластика, не пропускающего влагу. Слюда — отличный диэлектрик и выдерживает более высокие напряжения, чем бумага, не допуская дугового разряда между тарелки.Общие значения слюдяных конденсаторов колеблются примерно от 50 микрофарад, примерно до 0,02 мкФ. Показаны слюдяные конденсаторы. на рисунке 8-183.

Конденсаторы электролитические

Для емкостей более нескольких микрофарад площади пластин бумажные или слюдяные конденсаторы должны становиться очень большими; таким образом, электролитические конденсаторы обычно используются вместо них. Эти блоки обеспечивают большую емкость в небольшие физические размеры. Их значения колеблются от 1 до примерно 1500 микрофарад.В отличие от других типов, электролитические конденсаторы обычно поляризованы, и должен подвергаться воздействию постоянного напряжения или пульсирующего постоянного напряжения. Только; однако для использования используется специальный тип электролитического конденсатора. в моторах.

Электролитический конденсатор широко используется в электронных схемах и состоит из двух металлических пластин, разделенных электролитом. Электролит в контакте с отрицательной клеммой, в пастообразной или жидкой форме, содержит отрицательный электрод.Диэлектрик очень тонкий. пленка оксида, нанесенная на положительный электрод конденсатора. В положительный электрод, который представляет собой алюминиевый лист, сложен для достижения максимальной область. Конденсатор подвергается процессу формования во время производства, в котором через него проходит ток. Поток тока приводит к нанесение тонкого слоя оксида на алюминиевую пластину.

Близкое расположение отрицательного и положительного электродов приводит к сравнительно высокому значению емкости, но допускает большую вероятность пробоя напряжения и утечки электронов с одного электрода на Другие.

Используются два типа электролитических конденсаторов: (1) мокрый электролитический. и (2) сухие электролитические конденсаторы. В первом случае электролит жидкость и емкость должны быть герметичными. Этот тип всегда должен быть устанавливается в вертикальном положении.

Электролит установки сухого электролиза представляет собой пасту, содержащуюся в разделитель из абсорбирующего материала, такого как марля или бумага. Сепаратор не только удерживает электролит на месте, но и предотвращает короткое замыкание тарелки.Конденсаторы сухие электролитические изготавливаются как цилиндрические, так и прямоугольная форма блока и может содержаться как внутри картона, так и металлические крышки. Поскольку электролит не может пролиться, сухой конденсатор может монтироваться в любом удобном положении. Показаны электролитические конденсаторы. на рисунке 8-184.

Конденсаторы параллельно и последовательно

Конденсаторы могут быть объединены параллельно или последовательно для получения эквивалентных значения, которые могут быть либо суммой отдельных значений (параллельно) или значение меньше наименьшей емкости (последовательно).Фигура 8-185 показаны параллельные и последовательные соединения.

При измерении емкости используются две единицы: фарада и кулон. Как было определено ранее, фарад — это величина емкости. присутствует в конденсаторе, когда хранится один кулон электрической энергии на пластины, и на конденсатор подается один вольт. Один кулон это электрический заряд 6,28 миллиарда миллиардов электронов. Из этого видно, что

В A на рисунке 8-185 напряжение E — это то же самое для всех конденсаторов.Общий заряд Qt представляет собой сумму всех индивидуальные сборы, Q1, Q2 и Q3.

Используя основное уравнение для конденсатора,

Общий заряд равен Qt = CtE, где Ct — общая емкость. С общий заряд конденсаторов, включенных параллельно, является суммой отдельных заряды конденсаторов,

Qt = Q1 + Q2 + Q3.

Используя оба уравнения для полного заряда, получаем уравнение

CtE = C1E + C2E + C3E.

Разделив обе части этого уравнения на E, получим

Ct = C1 + C2 + C3.

Эта формула используется для определения общей емкости любого числа конденсаторов параллельно. При последовательном расположении (B на рис. 8-185) ток одинаков во всех частях цепи. Каждый конденсатор вырабатывает напряжение во время заряда, и сумма напряжений всех конденсаторы должны быть равны приложенному напряжению E. По уравнению конденсатора приложенное напряжение, E, равно общему заряду, деленному на общий емкость, или

Общий заряд Qt равен заряду любого из конденсаторов. потому что один и тот же ток течет у всех в течение одного и того же промежутка времени, и поскольку заряд равен току, умноженному на время в секундах (Qt = I х Т).Следовательно,

Qt = Q1 = Q2 = Q3,

и

, так как в цепи с последовательно включенными конденсаторами

Et = E1 + E2 + E3,

где E1, E2 и E3 — напряжения трех конденсаторов. потом

Разделив обе части уравнения на Qt, получим

Величина, обратная общей емкости любого количества конденсаторов. in series равно сумме обратных величин отдельных значений.

Параллельные конденсаторы объединяются по правилу, аналогичному правилу, используемому для объединения резисторы последовательно. Последовательные конденсаторы объединяются по правилу, аналогичному этому. для комбинирования параллельных резисторов.

При последовательном подключении двух конденсаторов C1 и C2 общая емкость дается уравнением:

Номинальное напряжение конденсаторов

При выборе или замене конденсатора для использования в конкретной цепи, необходимо учитывать следующее: (1) Требуемое значение емкости. и (2) величина напряжения, которому должен подвергаться конденсатор.Если напряжение, приложенное к пластинам, слишком велико, диэлектрик будет сломается и возникнет дуга между пластинами. Конденсатор тогда короткое замыкание, и возможное протекание через него постоянного тока может вызвать повреждение других частей оборудования. Конденсаторы имеют напряжение рейтинг, который нельзя превышать.

Рабочее напряжение конденсатора — это максимальное напряжение, которое может наносить равномерно без опасности перегорания дуги. Рабочее напряжение зависит от от (1) типа материала, используемого в качестве диэлектрика и (2) толщины диэлектрика.

Номинальное напряжение конденсатора является фактором при определении емкости. потому что емкость уменьшается с увеличением толщины диэлектрика. Конденсатор высокого напряжения с толстым диэлектриком должен иметь большую площадь пластины, чтобы иметь такую ​​же емкость, как и у аналогичного низкого напряжения конденсатор с тонким диэлектриком. Сила некоторых часто используемых диэлектрические материалы перечислены на рисунке 8-186. Номинальное напряжение также зависит от частоты, потому что потери и результирующий эффект нагрева, увеличиваются с увеличением частоты.

Конденсатор, который можно безопасно заряжать до 500 В постоянного тока, не может быть безопасно подвергается воздействию переменного или пульсирующего постоянного тока, действующее значение которого составляет 500 вольт. An переменное напряжение 500 вольт (среднеквадратичное значение) имеет пиковое напряжение 707 вольт, а конденсатор, к которому он приложен, должен иметь рабочее напряжение не менее 750 вольт. Конденсатор следует подбирать так, чтобы он работал напряжение, по крайней мере, на 50 процентов больше, чем максимальное подаваемое напряжение к нему.

Емкость Реактивное сопротивление

Емкость, как и индуктивность, противодействует протеканию тока. Это сопротивление называется емкостным реактивным сопротивлением и измеряется в омах. Символ емкостного реактивного сопротивления — Xc. Уравнение,

аналогичен закону Ома и уравнению для тока в индуктивном схема. Чем больше частота, тем меньше реактивное сопротивление. Следовательно емкостное реактивное сопротивление,

Проблема:

Предполагается последовательная цепь, в которой подаваемое напряжение составляет 110 вольт. при 60 гц, а емкость конденсатора 80.Найдите емкостное реактивное сопротивление и ток.

Решение:

Чтобы найти емкостное реактивное сопротивление, уравнение заменяется на фарады путем деления 80 на 1000000, поскольку 1 миллион микрофарад равен 1 фараду. Это частное равно 0,000080 фарада. Это заменено в уравнении и


Найдите текущий расход: =

Емкостные реактивные сопротивления последовательно и параллельно

При последовательном соединении конденсаторов полное реактивное сопротивление равно сумме индивидуальных реактивных сопротивлений.Таким образом,

Xct = (Xc) 1 + (Xc) 2

Суммарное реактивное сопротивление конденсаторов, подключенных параллельно, находится в таким же образом вычисляется полное сопротивление в параллельной схеме:

Фазы тока и напряжения в реактивных цепях

Когда ток и напряжение проходят через ноль и достигают максимального значения при в то же время ток и напряжение считаются синфазными (A на рисунке 8-187).Если ток и напряжение проходят через ноль и достигают максимальные значения в разное время, ток и напряжение считаются равными быть не в фазе. В цепи, содержащей только индуктивность, ток достигает максимальное значение позже напряжения, отставание от напряжения на 90 °, или четверть цикла (B на рисунке 8-187). В цепи, содержащей только емкость, ток достигает максимального значения перед напряжением и током опережает напряжение на 90 °, или на одну четверть цикла (C на рисунке 8-187).Насколько ток отстает или опережает напряжение в цепи, зависит от относительные величины сопротивления, индуктивности и емкости в схема.

Заряд, разделение пластин и напряжение

Dynamics Track
Наклонная плоскость
Импульс

Конденсатор
Пластина Sep
Пластина Sep / Вольт
Диэлектрики

Цепи
Закон Ом
Серия / Параллельный

Wave Tank
Частота / длина волны
Two Pt Interf.

Оптическая скамья
Refraction
Фокусное расстояние

Параллельный пластинчатый конденсатор

Заряд конденсаторов, разделение пластин и напряжение

Конденсатор используется для хранения электрического заряда. Чем большее напряжение (электрическое давление) вы прикладываете к конденсатору, тем больше заряда нагнетается в конденсатор. Кроме того, чем большей емкостью обладает конденсатор, тем больший заряд будет вызван данным напряжением.Это соотношение описывается формулой q = CV, где q — накопленный заряд, C — емкость, а V — приложенное напряжение.

Глядя на эту формулу, можно спросить, что бы произошло, если бы заряд оставался постоянным, а емкость изменялась. Ответ, разумеется, таков, что напряжение изменится! Это то, что вы будете делать в этой лаборатории.

Лабораторный конденсатор

Конденсатор с параллельными пластинами — это устройство, используемое для изучения конденсаторов.Это сводит к минимуму функцию конденсатора. Конденсаторы в реальном мире обычно скручены по спирали в небольших корпусах, поэтому конденсатор с параллельными пластинами значительно упрощает привязку функции к устройству.

Этот конденсатор работает, накапливая противоположные заряды на параллельных пластинах, когда напряжение подается с одной пластины на другую. Количество заряда, который перемещается в пластины, зависит от емкости и приложенного напряжения в соответствии с формулой Q = CV, где Q — заряд в кулонах, C — емкость в фарадах, а V — разность потенциалов между пластинами в вольт.

Конденсаторы накапливают энергию

Если напряжение подается на конденсатор, а затем отключается, заряд, накопленный в конденсаторе, сохраняется до тех пор, пока конденсатор каким-либо образом не разрядится. Между пластинами возникает электрическое поле, которое позволяет конденсатору накапливать энергию. Это один из полезных аспектов конденсаторов, способность накапливать энергию в электрическом поле, чтобы ее можно было использовать позже.

От чего зависит емкость?

Количество заряда, которое может храниться на один приложенный вольт, определяется площадью поверхности пластин и расстоянием между ними.Чем больше пластины и чем ближе они расположены, тем больше заряда может храниться на каждый вольт разности потенциалов между пластинами. Заряд, накопленный на приложенный вольт, представляет собой емкость, измеряемую в фарадах.

Может ли изменение емкости заряженного конденсатора изменить его напряжение?

Лабораторный конденсатор можно регулировать, поэтому мы можем провести интересный эксперимент с емкостью и напряжением. Если конденсатор имеет постоянный заряд, изменение емкости должно вызвать изменение напряжения.Раздвигание пластин приведет к уменьшению емкости, поэтому напряжение должно увеличиться.

Как можно математически определить емкость нашего конденсатора?
Для конденсатора с параллельными пластинами емкость определяется по следующей формуле:

С = ε 0 А / сут

Где C — емкость в Фарадах, ε 0 — постоянная диэлектрической проницаемости свободного пространства (8,85×10 -12), A — площадь пластин в квадратных метрах, а d — расстояние между пластинами в метрах.

Фарада — это очень большая величина емкости, поэтому мы будем использовать метрические префиксы для получения более удобных чисел. Емкость обычно измеряется в микрофарадах (мкФ), что составляет 1,0×10 -6F, или пикофарадах (пФ), что составляет 1,0×10 -12F. 1.0F = 1,000,000 мкФ = 1,000,000,000,000 пФ! Будьте очень внимательны с расчетами!

Этот расчет даст вам приблизительное значение емкости лабораторного конденсатора. Однако есть и другие факторы, которые вносят ошибки в реальные измерения емкости и напряжения.Вам нужно внимательно учитывать эти факторы.

Лабораторное оборудование:

Для получения хороших результатов эта лабораторная деятельность требует специального оборудования. Вам нужен хороший стабилизированный источник питания, чтобы напряжение, подаваемое на конденсатор, было одинаковым при каждом испытании.

Вам также нужен очень точный способ измерения напряжения между пластинами без резистивной нагрузки на конденсатор. Количество накопленного заряда очень мало, поэтому обычный вольтметр не подойдет.Мельчайший заряд, накопленный в конденсаторе, просто разрядится через счетчик, делая любые измерения бесполезными. Вы будете использовать специальный прибор для измерения напряжения, называемый электрометром, который измеряет напряжение без разряда конденсатора.

Одна из проблем электрометра заключается в том, что он имеет некоторую собственную емкость. Поскольку эта емкость параллельна емкости конденсатора, встроенная емкость выводов должна быть добавлена ​​к емкости конденсатора.

Назначение:

Целью данной лабораторной работы является исследование взаимосвязи между разделением пластин и напряжением в конденсаторе с параллельными пластинами, который поддерживается постоянным зарядом.

Оборудование:

  • Конденсатор переменной емкости
  • Электрометр
  • Регулируемый источник питания
  • Поводки для перемычек
  • Провода для электрометра

Осторожно:

Это хрупкое оборудование. Все должно сочетаться с легчайшими прикосновениями. Ничего не заставляйте!

Ваша первая задача — предсказать, что произойдет с напряжением конденсатора, когда вы зарядите его источником 10 В, а затем раздвинете пластины (что уменьшит емкость). Вы сделаете это в следующем разделе.

Теоретические расчеты:

Сначала необходимо рассчитать теоретическую емкость для каждого расстояния между пластинами. Мы сделаем первое, а потом вы сможете сделать все остальное! Самая сложная часть этого — правильно настроить юниты. Проще всего поставить все в метрах для расчетов:

  1. Измерьте диаметр пластин конденсатора в сантиметрах. Ваш размер должен быть около 17,8 см
  2. Разделите диаметр на 100, чтобы получить размер в метрах.Результат — 0,178 м. Разделите это на два, чтобы получить радиус: 0,089 м
  3. Площадь пластины определяется по общей формуле A = πr 2. Подставьте числа, чтобы получить A = π (0,089) 2 = 0,0249 м 2
  4. Преобразуйте расстояние между пластинами (1 мм) в метры, разделив на 1000. 1/1000 = 0,001 м.
  5. Используйте это число в формуле C = ε 0A / d, чтобы определить расчетную емкость, таким образом: C = 8,85×10 -12 (0,0249) / 0,001 = 2,20×10 -10. Это равно 220×10 -12F или 220pF
  6. Добавьте встроенную емкость электрометра (50 пФ) к теоретической емкости, чтобы получить 270 пФ.
  7. Запишите этот результат (270 пФ) в столбец «Расчетная емкость» и строку 1 мм.
  8. Повторите этот процесс для других расстояний между пластинами. Обратите внимание, что площадь пластины одинакова для всех, поэтому все, что вам нужно сделать, это повторить шаги 5, 6 и 7, вставляя правильные значения для интервала в каждом случае.
  9. Теперь вы рассчитаете теоретическое напряжение для каждого интервала. Предположим, что для шага 1,0 мм напряжение составляет 10 В, поэтому вы можете просто указать это значение в таблице.Во-первых, вы определяете количество заряда в конденсаторе при таком расстоянии и напряжении. Используйте формулу Q = CV, чтобы определить заряд, таким образом: Q = 270×10 -12F (10V) = 2700×10 -12C. Этот заряд остается неизменным на всех расстояниях между пластинами, поэтому вы можете ввести одно и то же значение во весь столбец Расчетный заряд! Теперь используйте это значение заряда, чтобы определить расчетное напряжение на всех других расстояниях. Например, при расстоянии 5 мм используйте формулу V = Q / C, таким образом: V = 2700×10 -12C / 94,0×10 -12F = 28,7V. Введите это значение в столбец «Расчетное напряжение» в строке 5 мм.
  10. Повторите тот же расчет напряжения для оставшихся расстояний между пластинами. Используйте рассчитанную емкость и постоянный заряд для каждого промежутка и введите значение напряжения в столбец «Расчетное напряжение» таблицы.
  11. Поздравляем! Вы закончили предварительные расчеты! Все, что вам нужно сделать сейчас, это провести фактические измерения!

В следующих разделах вы проведете реальный эксперимент для проверки (или, возможно, не проверки!) Ваших теоретических расчетов.

Процедура настройки переменного конденсатора (если лаборатория уже настроена, переходите к следующему разделу!)

  1. Поместите переменный конденсатор в середину лабораторного стола так, чтобы отметка 0 см находилась слева от вас. Не ставьте конденсатор слишком близко к краю стола!
  2. Разместите блок питания за конденсатором переменной емкости. Подключите блок питания, но не включайте его.
  3. Подключите красный и черный перемычки к красной и черной клеммам источника питания. Просто прикрепите зажим «крокодил» к отверстию и оставьте другой конец проводов свободным.
  4. Поместите электрометр слева от конденсатора.
  5. Присоедините плоские клеммы выводов электрометра к клеммам на задней стороне каждой пластины конденсатора. Красный провод идет к правой пластине, черный провод идет к левой пластине.
  6. Вставьте разъем BNC в электрометр.
  7. Установите пластины на расстоянии не менее 1 мм.Белые бамперы предотвращают сближение пластин. Если пластины не параллельны друг другу, используйте регулировочные ручки в центре правой опоры, чтобы выровнять пластины. Левый край пластикового язычка, выступающий в сторону шкалы, должен быть совмещен с отметкой 1 мм.

Сбор экспериментальных данных

  1. Убедитесь, что оборудование настроено правильно и полностью.
  2. Поверните все четыре регулятора на блоке питания против часовой стрелки до упора.
  3. Поверните крайнюю левую ручку (Fine Current) в положение на 12 часов (прямо вверх!)
  4. Включите источник питания. Дисплеи должны загореться.
  5. Используйте ручки Fine и Coarse Voltage (две крайние правые ручки), чтобы установить напряжение на 10,0 В.
  6. Установите пластины на минимальное значение
  7. Установите электрометр на шкалу 30 В.
  8. Нажмите кнопку питания на электрометре. Должен загореться светодиод 30 В.
  9. Нажмите кнопку нуля на электрометре. Это обнуляет счетчик и обеспечивает нулевое напряжение на пластинах относительно друг друга.
  10. На мгновение прикоснитесь к проводам от источника питания к пластинам, черный к левой пластине и красный к правой пластине.
  11. Электрометр должен показывать 12 В в этой точке (12 В — это первая маленькая отметка над «1» на нижней шкале. Если он не проверяет вашу настройку, попробуйте еще раз.Иногда вам нужно несколько раз прикоснуться проводами к пластинам, чтобы получить правильные показания 12 В.
  12. С этого момента вы должны быть осторожны, чтобы не прикасаться к пластинам. Прикоснувшись к ним, вы измените заряд в пластинах и испортите данные!
  13. Следите за электрометром, чтобы убедиться, что заряд сохраняется. Если вы видите падение напряжения более чем на вольт за 30 секунд, остановитесь и выясните, что не так, прежде чем продолжить.
  14. Переключите электрометр на настройку 100 В. Счетчик должен по-прежнему показывать 12 В, но по шкале 100 В.
  15. Осторожно раздвиньте пластины на расстояние 5 мм.
  16. Снимите показание электрометра и запишите его в таблицу под столбцом «Измеренное напряжение».
  17. Повторите два предыдущих шага для других расстояний между пластинами и запишите соответствующие данные.

Разделение пластин

(мм)

Расчетная емкость
(пФ)

Расчетный сбор

(пКл)

Расчетное напряжение

(В)

Измеренное напряжение

(В)

1

5

10

15

20

25

30

35

40

Анализ данных:

  1. На миллиметровой бумаге постройте расчетную емкость по оси x (горизонтальная) в зависимости от напряжения на оси y (вертикальная).Нанесите на график рассчитанное и измеренное значение напряжения, используя разные цвета или стили линий, чтобы различать две кривые. Убедитесь, что вы выбрали подходящие масштабы и четко обозначили оси и масштабы. Лучше всего ориентировать бумагу длинной осью в горизонтальном направлении («альбомный режим»).
  2. Изучите свой график и ответьте на следующие вопросы:

  1. Подтверждают ли ваши измеренные данные измеренные значения?
  1. Две кривые имеют одинаковую форму? Если да, то на что это указывает?

  2. Что бы вы сделали, чтобы повысить точность собираемых данных?
  3. Формула для энергии, запасенной в конденсаторе, U e = ½CV 2.Сохраняется ли энергия, запасенная в конденсаторе, постоянной при изменении расстояния между пластинами? Он идет вверх или вниз? Обсудите, откуда пришла или куда ушла энергия.

Емкость

  • • Что такое емкость?
  • • Диэлектрик.
  • • диэлектрическая проницаемость.
  • • Электрическая прочность и максимальное рабочее напряжение.
  • • Расчет заряда конденсатора.

Емкость

Количество энергии, которое может хранить конденсатор, зависит от величины или ЕМКОСТИ конденсатора. Емкость (символ C) измеряется в базовой единице FARAD (символ F). Один фарад — это величина емкости, которая может хранить 1 кулон (6,24 x 10 18 электронов), когда он заряжен до напряжения 1 вольт. Однако Фарада слишком большая единица для использования в электронике, поэтому следующие единицы емкости более полезны.

Дополнительный блок Аббревиатура Стандартное обозначение
мкФ мкФ х 10 -6
нано Фарады нФ х 10 -9
пик Фарад пФ х 10 -12

Помните, однако, что при решении задач, связанных с емкостью, формулы и используемые значения должны быть в основных единицах измерения: фарадах, вольтах и ​​т. Д.Поэтому при вводе значения 0,47 нФ, например, в формулу (или ваш калькулятор), его следует вводить в фарадах, используя версию стандартной формы для инженерных обозначений: 0,47 x 10 -9 (Загрузите буклет «Советы по математике», чтобы узнать больше Информация).

Емкость зависит от четырех вещей;

1. Площадь плит

2. Расстояние между пластинами

3. Тип диэлектрического материала

4. Температура

Из этих четырех наименьшее влияние на большинство конденсаторов оказывает температура.Стоимость большинства конденсаторов довольно стабильна в «нормальном» диапазоне температур.

Значения конденсатора могут быть фиксированными или переменными. Большинство переменных конденсаторов имеют очень маленькое значение (несколько десятков или сотен пФ). Значение может быть изменено на:

.
  • • Изменение площади пластин.
  • • Изменение толщины диэлектрика.

Емкость (C) ПРЯМО ПРОПОРЦИОНАЛЬНО ПЛОЩАДИ ДВУХ ПЛАСТИН , которые непосредственно перекрывают друг друга, чем больше площадь перекрытия, тем больше емкость.

Емкость ОБРАТНО ПРОПОРЦИОНАЛЬНО РАССТОЯНИЮ МЕЖДУ ПЛАСТИНАМИ. т.е. если пластины раздвигаются, емкость уменьшается.

Диэлектрик

Электроны на одной пластине конденсатора влияют на электроны на другой пластине, вызывая искажение орбит электронов внутри диэлектрического материала (изолирующего слоя между пластинами). Величина искажения зависит от природы диэлектрического материала и измеряется диэлектрической проницаемостью материала.

Разрешение

Проницаемость указывается для любого конкретного материала как ОТНОСИТЕЛЬНАЯ ДОПУСТИМОСТЬ, которая является мерой эффективности диэлектрического материала. Это число без единиц, которое указывает, насколько диэлектрическая проницаемость материала больше, чем диэлектрическая проницаемость воздуха (или вакуума), для которого задана диэлектрическая проницаемость 1 (единица). Например, если диэлектрический материал, такой как слюда, имеет относительную диэлектрическую проницаемость 6, это означает, что конденсатор будет иметь диэлектрическую проницаемость, а значит, и емкость, в шесть раз больше, чем у конденсатора с такими же размерами, но диэлектриком которого является воздух.

Диэлектрическая прочность

Другой важный аспект диэлектрика — ДИЭЛЕКТРИЧЕСКАЯ ПРОЧНОСТЬ. это указывает на способность диэлектрика выдерживать напряжение, приложенное к нему, когда конденсатор заряжен. В идеале диэлектрик должен быть как можно тоньше, чтобы обеспечить максимальную емкость для данного размера компонента. Однако чем тоньше диэлектрический слой, тем легче разрушаются его изоляционные свойства. Таким образом, диэлектрическая прочность определяет максимальное рабочее напряжение конденсатора.

Максимальное рабочее напряжение (VDCwkg max)

При использовании конденсаторов очень важно, чтобы максимальное рабочее напряжение, указанное производителем, не превышалось. В противном случае существует большая опасность внезапного пробоя изоляции внутри конденсатора. Поскольку в это время на конденсаторе существует максимальное напряжение (отсюда и пробой), большие токи будут протекать с реальным риском возгорания или взрыва в некоторых цепях.

Заряд конденсатора.

Заряд (Q) конденсатора зависит от комбинации вышеперечисленных факторов, которые можно представить вместе как емкость (C) и приложенное напряжение (V). Для компонента данной емкости соотношение между напряжением и зарядом является постоянным. Увеличение приложенного напряжения приводит к пропорциональному увеличению заряда. Эту связь можно выразить формулой;

Q = CV

или

C = Q / V

или

V = Q / C

Где V — приложенное напряжение в вольтах.

C — емкость в Фарадах.

Q — количество заряда в кулонах.

Итак, любое из этих количеств может быть найдено, если известны два других. Формулы можно легко переставить, используя простой треугольник, аналогичный тому, который используется для расчета закона Ома при проведении расчетов резисторов.

Практические задачи: решения емкости — Physics-prep.com

Практические проблемы: решения конденсаторов

1.(легко) Определите количество заряда, накопленного на любой пластине конденсатора (4×10 -6 F) при подключении к 12-вольтовой батарее.
C = Q / V
4×10 -6 = Q / 12
Q = 48×10 -6 C

2. (легко) Если расстояние между пластинами конденсатора составляет 2,0×10 -3 м, определите площадь пластин, если емкость равна точно 1 F.
C = ε o A / d
1 = ( 8,85×10 -12 ) A / (2,0×10 -3 )
A = 2,3×10 8 м 2

3.(умеренное) Рассчитайте напряжение батареи, подключенной к конденсатору с параллельными пластинами с площадью пластин 2,0 см 2 и расстоянием между пластинами 2 мм, если заряд, накопленный на пластинах, составляет 4,0 пКл.
Площадь = 2,0 см 2 (1 м / 100 см) 2 = 2,0×10 -4 м 2
C = ε o A / d
C = (8,85×10 -12 ) ( 2,0×10 -4 ) / (2,0×10 -3 )
C = 8,85×10 -13
C = Q / V
8,85×10 -13 = 4.0×10 -12 / В
В = 4,5 В

4. (легкий) Конденсатор с параллельными пластинами состоит из металлических пластин, каждая площадью 0,2 м 2 . Емкость 7,9 нФ. Определите расстояние между пластинами.
C = ε o A / d
7,9×10 -9 = 8,85×10 -12 (0,2) / d
d = 2,2×10 -4 m = 0,22 мм

5. (легкий) Конденсатор (параллельная пластина) заряжается батареей постоянного напряжения. Как только конденсатор достигает максимального заряда, аккумулятор удаляется из цепи.Опишите любые изменения, которые могут произойти в перечисленных здесь количествах, если пластины будут сдвинуты ближе друг к другу.
а. Заряд (Заряд, отложенный на пластинах, не меняется, когда аккумулятор извлекается, и, таким образом, заряд и плотность заряда остаются такими же, когда пластины перемещаются ближе друг к другу.)
b. Емкость (поскольку емкость составляет C = ε А / день, а площадь не меняется, любое уменьшение расстояния между пластинами (d) приведет к увеличению емкости.)
c.Напряжение (поскольку C = Q / V и заряд не меняется, увеличение емкости означает уменьшение напряжения.)
d. E-поле (Поскольку ΔV = -Ed, E-поле останется таким же, как и напряжение, и расстояние уменьшатся пропорционально.)

6. (умеренно) Чипы оперативной памяти используются в компьютерах для хранения двоичной информации в виде «единиц» и «нулей». Один из распространенных способов сохранить «единицу» — зарядить очень маленький конденсатор. Конечно же, тот же конденсатор без заряда представляет собой «ноль».Микросхема памяти содержит миллионы таких конденсаторов, каждый из которых соединен с транзистором (который действует как переключатель), образуя «ячейку памяти». Типичный конденсатор в ячейке памяти может иметь емкость 3×10 -14 F. Если напряжение на конденсаторе, показывающее «единицу», составляет 0,5 В, определите количество электронов, которые должны двигаться по конденсатору для его зарядки.
C = Q / V
3×10 -14 = Q / (0,5)
Q = 1,5×10 -14 C
# электроны = Полный заряд / заряд на один электрон
# электронов = 1.5×10 -14 / 1,6×10 -19
#electrons = 93750 электронов

7. (легко) C 1 = 10 F и C 2 = 5 F. Определите эффективную емкость для C 1 и C 2 , соединенных последовательно и параллельно.
Последовательно:
1 / C = 1 / C 1 + 1 / C 2
1 / C = 1/10 + 1/5
C = 3,3 F
Параллельно:
C = C 1 + C 2
C = 10 + 5 = 15 F

8. (умеренно) Если два рассматриваемых конденсатора №7 были подключены к 50-вольтовой батарее, определите напряжение на конденсаторах для каждого типа подключения.
Для последовательного соединения:
Заряд каждого конденсатора равен заряду эффективной емкости.
C = Q / V
3,3 = Q / 50
Q = 165 C
Для конденсатора 10 Ф:
10 = 165 / В
В = 17 В
Для конденсатора 5 Ф:
5 = 165 / В
В = 33 В
Для параллельного подключения:
Напряжение на каждом конденсаторе одинаковое (50 В).

9. (средний) Оцените схему, показанную ниже, чтобы определить эффективную емкость, а затем заряд и напряжение на каждом конденсаторе.

Эквивалентная емкость 4 мкФ. Напряжение на эквивалентном конденсаторе составляет 20 вольт.
Это напряжение также присутствует на обоих конденсаторах по 2 мкФ, которые были созданы последовательными комбинациями в каждой ветви.
Найдите заряд на каждом конденсаторе 2 мкФ:
C = Q / V
2 мкФ = Q / 20
Q = 40 мкК
Конденсаторы 4 мкФ в каждой ветви имеют такой же заряд, как и конденсаторы 2 мкФ. Используйте это, чтобы найти напряжение на каждом из них:
C = Q / V
4 мкФ = 40 мкКл / В
V = 10 вольт
Таким образом, каждый из исходных конденсаторов 4 мкФ имеет заряд 40 мкКл и напряжение 10 вольт.

10. (средний) Оцените схему, показанную ниже, чтобы определить эффективную емкость, а затем заряд и напряжение на каждом конденсаторе.

Эффективная емкость составляет 6 мкФ при напряжении 100 В.
Напряжение на конденсаторах 4 мкФ и 2 мкФ также равно 100 В.
Заряд на конденсаторе 4 мкФ:
C = Q / V
4 мкФ = Q / 100
Q = 400 мкКл
Заряд конденсатора 2 мкФ:
C = Q / V
2 мкФ = Q / 100
Q = 200 мкКл
Все три конденсатора по 6 мкФ также имеют заряд 200 мкКл.
Найдите напряжение для конденсаторов 6 мкФ:
C = Q / V
6 мкФ = 200 мкКл / В
V = 33,3 В

11. (умеренное) Оцените схему, показанную ниже, чтобы определить эффективную емкость, а затем заряд и напряжение на каждом конденсаторе.

Эквивалентная емкость 6 мкФ. Напряжение на эквивалентной емкости равно 40 В, как и напряжение на конденсаторах 3 мкФ, и такое же, как на конденсаторах 1 мкФ и 2 мкФ.
Найдите заряд конденсатора 1 мкФ:
C = Q / V
1 мкФ = Q / 40
Q = 40 мкК
Найдите заряд конденсатора 2 мкФ:
C = Q / V
2 мкФ = Q / 40
Q = 80 мкКл
Найдите заряд конденсаторов 3 мкФ:
C = Q / V
3 мкФ = Q / 40
Q = 120 мкКл
Это одинаковый заряд на каждом из конденсаторов 6 мкФ.
Найдите напряжение на каждом из конденсаторов 6 мкФ:
C = Q / V
6 мкФ = 120 мкКл / В
V = 20 В

Что такое емкость? — Последовательная и параллельная схема измерения емкости

Емкость определяется как способность элемента накапливать электрический заряд. Конденсатор накапливает электрическую энергию в виде электрического поля двумя электродами конденсатора, один из которых является положительным, а другой — отрицательным.

Другими словами, емкость — это мера заряда на единицу напряжения, которая может храниться в элементе.Он обозначается (C), а его единица измерения — Фарад (F).

Состав:

Емкость в основном подразделяется на два типа; это собственная емкость и взаимная емкость. Вещество с большей собственной емкостью накапливает больше электрических зарядов, а вещество с низкой емкостью накапливает меньше электрических зарядов.

Объяснение и определение емкости

Если две параллельные пластины соединены, перекрывают друг друга и подключены к источнику постоянного напряжения, как показано на рисунке.Две пластины разделены изолирующим диэлектриком, поэтому заряды не пересекают друг друга. Один вывод параллельной пластины подключен к положительному источнику питания, а другой — к отрицательному. Когда источник питания включен, конденсатор начинает заряжаться и накапливает энергию, даже если источник питания отключен.

Уравнение емкости определяется следующим образом:

Где,

  • C — емкость в Фарадах или Микрофарадах
  • A — площадь перекрытия двух пластин в квадратном метре
  • d — расстояние между двумя пластинами в метрах
  • ε 0 известна как электрическая постоянная
  • ε r — диэлектрическая проницаемость материала между двумя пластинами

Емкость считается равной одной фараде, если один кулон заряда сохраняется с одним вольт на двух электродах элемента.Емкостной элемент называется конденсатором.

Заряд конденсатора в любой момент времени

q — это количество заряда, которое может храниться в конденсаторе емкости (C) против разности потенциалов (v) вольт.

Где i, q и v представляют собой мгновенное значение тока, заряда и напряжения соответственно.

Где

v 0 — начальное напряжение конденсатора

v t конечное напряжение конденсатора

Сейчас,

Мощность, потребляемая конденсатором, определяется уравнением, приведенным ниже.

Энергия, запасенная конденсатором, определяется как

Ток через конденсатор равен нулю, если приложенное напряжение на конденсаторе постоянное.это означает, что когда на конденсатор подается постоянное напряжение без начального заряда, конденсатор сначала действует как короткое замыкание, но как только он полностью заряжается, конденсатор начинает вести себя как разомкнутая цепь.

Конденсатор только накапливает энергию и никогда не рассеивает ее ни в какой форме. Он может хранить конечное количество энергии, даже если ток через конденсатор равен нулю.

Типы конденсатора

Различают следующие типы конденсаторов:

  • Бумажный конденсатор
  • Конденсатор воздушный
  • Пластиковый конденсатор
  • Конденсатор серебряный слюдяной
  • Конденсатор керамический
  • Конденсатор электролитический
  • Конденсатор фарфоровый
Серия

и параллельная емкость в цепи

Цепь конденсатора серии

Если количество конденсаторов, например, C 1 , C 2 , C 3 ….., соединенные последовательно, называется последовательной конденсаторной цепью. Ток, протекающий в цепи этого типа, будет одинаковым на всех конденсаторах, поскольку они соединены последовательно. Последовательное соединение конденсатора показано ниже на рисунке:

Эквивалентная емкость определяется уравнением:


Параллельная цепь конденсатора

Если несколько конденсаторов соединены друг с другом, как при параллельном соединении, цепь называется параллельной цепью конденсаторов.Схема показана ниже:

Эквивалентная емкость в параллельной цепи определяется уравнением, показанным ниже:

Это все о емкости.

Емкость и конденсатор Вопросы и ответы

Этот набор вопросов и ответов с множественным выбором (MCQ) по базовой электротехнике посвящен теме «Емкость и конденсатор».

1. Конденсатор — это устройство, используемое для __________
a) хранения электроэнергии
b) изменения сопротивления
c) хранения магнитной энергии
d) рассеивания энергии
Посмотреть ответ

Ответ: a
Объяснение: Конденсатор используется для хранения заряда .Он хранит электрическую энергию между пластинами.

2. Какой тип энергии накапливает конденсатор?
a) кинетическая энергия
b) колебательная энергия
c) потенциальная энергия
d) тепловая энергия
Посмотреть ответ

Ответ: c
Пояснение: Конденсатор сохраняет заряд между пластинами. Этот заряд является стационарным, поэтому можно сказать, что конденсатор хранит потенциальную энергию.

3. Блоки конденсаторов __________ по прошествии длительного времени.
a) переменный ток
b) постоянный ток
c) как переменный, так и постоянный ток
d) ни переменный, ни постоянный ток
Посмотреть ответ

Ответ: b
Пояснение: Конденсатор блокирует постоянный ток в установившемся состоянии и пропускает переменный ток.

4. Почему конденсаторный блок подает сигнал постоянного тока в установившемся состоянии?
а) из-за высокой частоты сигнала постоянного тока
б) из-за нулевой частоты сигнала постоянного тока
в) конденсатор не пропускает ток в установившемся состоянии
г) из-за нулевой частоты сигнала постоянного тока
Посмотреть ответ

Ответ: d
Пояснение: Частота сигнала постоянного тока равна нулю. Таким образом, емкостное реактивное сопротивление X C = 1 / 2πfc становится бесконечным, и конденсатор ведет себя как разомкнутая цепь для сигнала постоянного тока. Следовательно, сигнал постоянного тока конденсаторного блока.

5. Если конденсатор с параллельными пластинами площадью 2 м 2 и расстоянием между пластинами 1 м сохраняет заряд 1,77 * 10 -11 C. Какое напряжение на конденсаторе?
a) 1V
b) 2V
c) 3V
d) 4V
Просмотреть ответ

Ответ: a
Объяснение: C = € 0 A / d
Подставляя значения d, A, мы получаем C = 2 € 0 .
Q = CV
V = 1 В.

6. Что из перечисленного является пассивным?
a) Транзистор
b) Выпрямитель
c) Конденсатор
d) Вакуумные трубки
Посмотреть ответ

Ответ: c
Объяснение: Конденсатор является пассивным устройством, поскольку он потребляет энергию, остальные генерируют энергию, поэтому они являются активными устройствами.

7. Каково значение емкости конденсатора с напряжением 4 В и зарядом 16 C?
a) 2F
b) 4F
c) 6F
d) 8F
Посмотреть ответ

Ответ: b
Пояснение: Q прямо пропорционально V. Константа пропорциональности в этом случае равна C, то есть емкости. Следовательно, Q = CV. Из соотношения C = Q / V = ​​16/4 = 4F.

8. Для какой средней емкости высокая?
a) Воздух
b) Слюда
c) Вода
d) Металл
Посмотреть ответ

Ответ: d
Пояснение: Предполагается, что металлы имеют высокое значение диэлектрической проницаемости, поэтому они обладают высокой емкостью.

Sanfoundry Global Education & Learning Series — Основы электротехники.

Чтобы практиковать все области базовой электротехники, вот полный набор 1000+ вопросов и ответов с несколькими вариантами ответов .

Примите участие в конкурсе сертификации Sanfoundry, чтобы получить бесплатную Почетную грамоту. Присоединяйтесь к нашим социальным сетям ниже и будьте в курсе последних конкурсов, видео, стажировок и вакансий!

.

Добавить комментарий

Ваш адрес email не будет опубликован. Обязательные поля помечены *